Med Surg 2 Repro

¡Supera tus tareas y exámenes ahora con Quizwiz!

Black women, vitamin d deficiency, reproductive tract infection, early period, red meat, alcohol, hypertension

7 risk factors of uterine leiomyomas

excision, cryoablation, electrocautery, ultrasonic aspiration, carbon dioxide laser, surgical removal, intra-lesion interferon injections

7 surgical managements of genital warts

HIV infection

transgender women are at higher risk of

testicular torsion

twisting of the spermatic cord. Blocks arterial flow in and venous flow out

phimosis

uncircumsized male

cryptorchidism

undescended testicles

cryodestruction, immune-mediated therapies

2 nonsurgical managements for genital warts

anterior colporrhaphy, posterior colporrhaphy

2 surgical treatments for pelvic organ prolapse

viral culture, direct fluorescent antibody, polymerase chain reaction assays of lesions

3 assessment test for herpes

Transrectal US, cystoscopy, Bladder ultrasound

3 diagnostic assessments for BPH

U&A, CBC, BUN, PSA, Serum acid phosphatase, biopsy, C&S

7 labs to test for BPH

B

A 22-year-old client wants to discuss options for treatment for gender dysphoria. He states "I'm confused and I need to talk to somebody, but I don't know what to do and who to talk to. I don't want my parents to know. Can you help me?" What is the nurse's priority response? A. "It depends on what kind of insurance you have." B. "What you say here will be confidential?" C. "Let's see if we can get a therapist to see you." D. "I can make you an appointment with the provider."

D

A 68-year-old client has recently undergone a prostate biopsy. Which assessment finding will the nurse report to the health care provider? A. Rust-colored semen B. Slight rectal bleeding C. Mild pain and soreness at the site D. Temperature of 101.6° F (38.7° C)

C

A nurse is assisting with the care of a client who has human immunodeficiency virus (HIV). Which of the following types of isolation should the nurse implement to prevent the transmission of HIV? A. Protective isolation B. Droplet precautions C. Standard precautions D. Airborne precautions

Within 6 months

A person will get a positive hiv test how soon

VERY CONTAGIOUS

A syphillis rash is

Myolysis

Ablation of tissue with uterine leiomyomas

Pelvic organ prolapse

Abnormal uterine bleeding, watery/bloody discharge, pain in lower abdomen and back, palpable uterine mass

Genital Herpes

Acute, recurring, common viral disease. Two types: HSV-1, HSV-2

ANS: A, C, D, F In the documentation of a breast mass, skin changes such as dimpling (peau d'orange), nipple retraction, and whether the mass is fixed or movable are charted. The location of the mass should be stated by the "face of a clock." Skin ulceration is also a common sign. Dense breast tissue and nontender axillary nodes are not abnormal assessment findings that may indicate breast cancer.

After a breast examination, the nurse is documenting assessment findings that indicate possible breast cancer. Which abnormal findings need to be included as part of the client's electronic medical record? (Select all that apply.) a. Peau d'orange b. Dense breast tissue c. Nipple retraction d. Mobile mass at 2 o'clock e. Nontender axillary nodes f. Skin ulceration

C, D

After returning from transurethral resection of the prostate, the client's urine in the continuous bladder irrigation system is a burgundy color. Which client needs does the nurse anticipate will be prescribed after the surgeon sees the client? (Select all that apply.) Select all that apply. A. Encouragement of oral fluids B. Emergency surgery C. Monitoring for anemia D. Antispasmodic drugs E. Increased intermittent irrigation

C

An 18-year-old female is diagnosed with possible toxic shock syndrome (TSS) and has these vital signs: T 103.2° F (39.6° C), P 124 beats/min, R 36 breaths/min, BP 84/30 mm Hg. Which primary health care provider request does the nurse implement first? A. Begin infusion of normal saline IV B. Obtain blood cultures × 2 sites. C. Administer corticosteroids as prescribed. D. Give cefazolin as prescribed.

ureterovaginal fistula

An abnormal opening Between ureter and vagina

B

An older adult client reports uncomfortable sexual intercourse associated with vaginal dryness. Which nursing response is appropriate? A. "You may need to have additional pelvic examinations." B. "Products such as water-soluble lubricants may be helpful." C. "Be sure to tell your primary health care provider about this." D. "Let me teach you how to do Kegel exercises."

benign

Assess _ lumps as mobile and round or oval

Cystocele

Bladder that protrudes through vaginal wall

secondary syphillis

Can be mistaken for the flu, this stage is systemic with syphillis

Endometriosis

Can causes scarring and adhesions in the pelvis, also can cause cysts

Atypical hyperplasia

Can lead to breast cancer, don't use contraceptives due to increased risk

Fibrocystic changes and cysts

Causes by an imbalance to estrogen to progesterone ratio

genital warts

Certain types of _ can be associated with precancerous or dysplastic lesions and cancer

Fibrocystic changes and cysts

Changes in response to hormones

cART

Combined antiretroviral treatment after exposure to HIV.

Dermoid

Cyst on the reproductive tissue

Cystadenoma

Cyst on the surface of the ovary that can get large

corpus luteum

Cyst that forms after the egg leaves, the opening to cysts gets blocked

Follicular cyst

Cyst that forms when egg does not leave the follicle

Endometriosis

Cysts from endometriosis

Ovarian cysts

Develops regularly in a woman and does not cause problems

void

Drug therapy for patients with BPH will make it easier for patients to

ANS: D A dusky color of the breast flap could indicate poor tissue perfusion. The nurse would notify the primary health care provider to preserve the tissue. It is normal to have a slightly reddened incision as the skin heals. The blood pressure is within normal limits and the temperature is slightly elevated but would be monitored.

During dressing changes, the nurse assesses a client who had breast reconstruction. Which finding would cause the nurse to take immediate action? a. Slightly reddened incisional area b. Blood pressure of 128/75 mm Hg c. Temperature of 99° F (37.2° C) d. Dusky color of the breast flap

Lower immunity

Endocrine complications with HIV cause

AIDS

Everyone with stage 3 HIV has

Uterine Leiomyomas

Excessive local growth of smooth muscle layers

A

For the 12 months prior to surgery, what is one of the requirements for a client requesting a vaginoplasty or a phalloplasty? A. Continuously living in the role of the desired gender identity B. Initiation of hormone therapy C. Monthly vocal coaching D. Therapy sessions with a licensed psychotherapist

B

Four women phone the gynecology clinic about having new-onset of spotty vaginal bleeding. Which call does the RN decide to return first? A. A 34 year old with a history of multiple leiomyomas B. A 65 year old with no previous gynecologic problems C. A 48 year old who had an endocervical curettage yesterday D. A 23 year old using medroxyprogesterone acetate

D

From what age may a child begin to feel a sense of maleness or femaleness? A. 6 months B. 5 years C. Birth D. 2 years

tertiary syphillis

Gummatous lesions on skin, bones, internal organs. Cardiovascular infection, progressive dementia and ataxia are signs and symptoms of

Breakdown of skin

HIV can also cause

Diarrhea

HIV can cause what bowel problems

Prophylaxis-

Have liver and kidney tests done every 3 months with

28 days

How long do you take cART for

B

How will the nurse refer to a client who self-identifies as the opposite gender? A. A transgender B. Transgender individual C. Transvestite D. Transgendered person

gynecomastia

Spironolactone can cause

Cognitive decline

Stage 3 of HIV causes

vulvovaginitis

Symptoms include itching, change in vaginal discharge, odor, or lesions. Not considered an STD, yeast infections are considered this also

vulvovaginitis

Inflammation of lower genital tract resulting from disturbance of the balance of hormones and flora in vagina and vulva

Immune system

Malignancies lower

Truvada

Medication to decrease exposure or transmission. For people who are high risk for HIV

cART

Medication to prevent becoming positive after exposure

vaginoplasty

Monitor for potentially life-threatening complications of _, such as fistula development, bleeding, and wound infection after surgery

Uterine Leiomyomas

Most commonly diagnosed pelvic tumor

Deep needle sticks

Needle pricks do not cause transmission, what does?

Fibrocystic changes and cysts

No Proliferative breast lesion

BPH (benign prostatic hyperplasia)

Over 80% of Men older than 80 are at risk for

tumor development

Patients with cryptorchidism are at increased risk for

Transmission

Pregnant people with hiv need to stay on treatment to prevent

uterine artery embolization

Procedure that Starves blood flow to fibroid with uterine leiomyomas

Atypical hyperplasia

Proliferative breast lesion with atypia

Fibroadenoma

Proliferative breast lesion without atypia

Men

Prophylaxis is only for

cancer

Teach men who are uncircumcised about the importance of keeping the penis clean to prevent penile _

Rectocele

Rectum that protrudes through vaginal wall

cancer

Serum acid phosphatase looks for

genital warts

Small papillary growths that may grow into large cauliflower like masses

cancer

Teach patients to eat a well-balanced diet including fish, fruits, and vegetables to help prevent prostate _

15lb

Teach patients to not lift more than _ after open prostate surgery

urological emergency

Testicular torsion is a

A, D

The community health nurse is providing education to a group of women about risks for breast cancer. Which factor will the nurse include in the education session? (Select all that apply.) Select all that apply. A. High breast density B. First child at age 25 C. Male with gynecomastia D. Nulliparity E. Middle-age woman

D

The emergency nurse is assigned to care for four clients. Which client does the nurse see first? A. Young adult with a swollen, painful scrotum who has a recent history of mumps infection B. Older adult with a history of benign prostatic hyperplasia and palpable bladder distention C. Middle-age adult discharged 2 days ago after a TURP with hematuria D. Adolescent with an erection for "10 or 11 hours" who is reporting severe pain

A

The health care provider has completed a cervical biopsy on a client. Which postprocedure teaching will the nurse provide? A. "Use the antiseptic solution rinses to clean your perineum." B. "Abstain from intercourse for 24 hours after the procedure." C. "Rest for at least 12 hours after the procedure." D. "There is no limit on activity or weight-lifting."

More infections

The higher the viral load in HIV =

A

The human papilloma virus (HPV) test may be collected at the same time as the Papanicolaou (Pap) test for screening. Which finding indicates the highest risk for development of cervical cancer? A. Abnormal Pap results and positive HPV test B. Normal Pap results and no HPV infection C. Abnormal Pap results and no HPV infection D. Normal Pap results and positive HPV results

D

The nurse at the gynecology clinic is examining a woman's breasts. Which assessment finding requires immediate notification of the primary health care provider? A. Backache and breast fungal infection B. Ill-defined painful rubbery lump in the outer breast quadrant C. A 1-cm freely mobile rubbery mass discovered by the client D. Nipple discharge and dimpling

D

The nurse is providing education for a client who is experiencing gender dysphoria. What client statement reflects that further nursing teaching is needed? A. "I could live as the other sex full time if I want to." B. "Surgery is an option to change my breasts and face." C. "Talking to a psychotherapist might help me understand my identity." D. "I plan to buy my hormones on the internet."

D

The nurse is reviewing a laboratory report that indicates a decrease in a client's estradiol level. How does the nurse interpret this information? A. The client is experiencing a normal pregnancy. B. The client may have a malignant tumor. C. The client may be in menopause. D. The client may be pregnant with twins.

ANS: A The first few inches (centimeters) of the catheter must be washed daily starting at the penis and washing outward with soap and water. The other options are correct for self-management of a urinary catheter in the home setting.

The nurse is teaching an uncircumcised 65-year-old client about self-management of a urinary catheter in preparation for discharge to his home. What statement indicates the client needs more information? a. "I have to wash the outside of the catheter once a day with soap and water. b. "I should take extra time to clean the catheter site by pushing the foreskin back." c. "The drainage bag needs to be changed at least once a week and as needed." d. "I should pour a solution of vinegar and water through the tubing and bag."

antiviral

Treat severity, promote healing, decrease frequency of recurrent outbreaks that are not curative with herpes

Uterine Leiomyomas

Uterus can be taken out with

Genital warts

Very common viral disease, caused by certain types of HPV

Impaired gas exchange

What does HIV do to o2

Shrinks the fibroid down to a manageable level

What does hormone therapies due to uterine leiomyomas

8

What is the percentage of risk to transfer to baby

B

What teaching does the nurse provide to a client prior to a mammogram? A. "Do not eat anything for 12 hours before having a mammogram." B. "You must not wear deodorant the day of your mammogram." C. "You will not feel any discomfort during the mammography procedure." D. "A mammogram will x-ray the hard tissue of your breasts."

genital warts

_ may disappear and resolve, or recur

vesicovaginal fistula

abnormal opening between the bladder and the vagina

colovaginal fistula

abnormal opening between the colon and vagina

rectovaginal fistula

abnormal opening between the vagina and rectum

urethrovaginal fistula

abnormal opening between the vagina and the urethra

condyloma acuminatum

another term for genital warts

malignant

assess possible _ lumps as fixed and irregularly shaped

PID

rest in semi-fowlers position with

25

secondary syphillis develops in _% of untreated individuals within a few months

PID

cervical, urethral, and rectal specimens are used to assess for

painless

chancres that develop with primary syphillis are

chlamydia

doxycycline treats which std

benzathine penicillin G

drug therapy for syphillis

infertility in males

if gonorrhea causes epididymitis it can lead to

Erectile dysfunction

inability of an adult male to achieve an erection; impotence

2-20 days

incubation with genital herpes is how long

Chlamydia trachomatis

intracellular bacterium, causitive agent of cervitis, urethritis, proctitis, often asymptomatic

PID

look for a hx of sexual trauma and abuse with

PID

lower abdominal pain, irregular bleeding, dysuria, vaginal discharge, dyspareunia, malaise, fever, chills

PID

may need surgery if no response to other treatment with

injected ceftriaxone

medication treatment for gonorrhea

Chlamydia

men will have discharge with

90%

patients with HIV need to adhere to prescribed drug therapy regimen _% of the time

PID

pelvic exam may show discharge, or friable cervix with

D

A nurse is contributing to the plan of care for a client with AIDS who has developed stomatitis. Which of the following interventions should the nurse recommend for the plan of care? A. Rinse the mouth with chlorhexidine solution every 2 hours B. Limit fluid intake with meals C. Provide oral hygiene with a firm-bristled toothbrush after each meal D. Avoid salty foods

A

A nurse is reinforcing teaching with a client who is HIV positive about the early manifestations of acquired immune deficiency syndrome (AIDS). Which of the following statements should the nurse include in the teaching? A. "You can expect a persistent fever and swollen glands." B. "You can expect an elevated white blood cell count." C. "You can expect an increase in blood pressure and edema." D. "You can expect weight gain."

A

A client with erectile dysfunction has been prescribed a PDE-5 inhibitor. Which client statement demonstrates that nursing teaching about the medication has been effective? A. "I have to be sure to be careful when I sit up or stand up." B. "I can drink a little bit of alcohol before intercourse to help me out." C. "I will have my partner take this medication about 30 minutes before intercourse." D. "I need to increase fiber in my diet to avoid constipation."

genital herpes, chlamydia, gonorrhea, syphillis, canchroid, HIV

6 notifiable STIs

pap smear, blood test, HIV Test, biopsy, cultures for chlamydia and gonorrhea, screening for HPV and cervical dysplasia

6 tests for genital warts

malaise, low grade fever, headache, muscle aches, sore throat, adenopathy, joint pain

7 flu like symptoms of secondary syphillis

ANS: A Clients with HPV should be fully screened for other STIs since co-infection is common. Over the counter treatments should not be applied to genital tissue. HPV is not reportable. Contraception is not related.

A 24 year-old female has been diagnosed with genital warts. Which action by the nurse is best? a. Encourage the client to complete STI screening. b. Recommend an over-the-counter wart treatment for genital tissue. c. Report the case to the Centers for Infection Control and Prevention (CDC). d. Discuss popular options for contraception.

Myomectomy, hysterectomy

Two surgical interventions for uterine leiomyomas

Stage 1

Which stage of HIV has flulike symptoms

3 months

testis should drop into the testiles within _ months of birth

mastectomy

the arm of the surgical _ side should not be used for blood pressures, blood drawing, IV therapy, or injections.

MSM

with chlamydia, there is a high prevalence of rectal and pharyngeal infection in

week

with gonorrhea, first symptoms occur within a _ after sexual contact with an infected person

chancre

what is the first sign of primary syphillis

prostaglandin E1

With ED you will insert penile self injection with

gynecomastia

When men develop Breast tissue

fallopian tubes

what is the most common place PID may spread

uterus, fallopian tubes, adjacent pelvic structures

what three places may PID spread

3 way foley

what type of catheter will a patient with BPH have for home management

PID

when organisms move from endocervix upward through uterine cavity into fallopian tubes, multiple pathogens

chlamydia

which STD can impact reproduction

PID

which std can have systemic symptoms, can lead to sepsis if left untreated

serology testing

which type of test can differentiate between type 1 and 2 genital herpes

women

Who is more at risk for STD

A

A nurse is reinforcing dietary teaching with a client who has AIDS and stomatitis of the mouth. Which of the following instructions should the nurse include in the teaching? A. "You can suck on popsicles to numb your mouth." B. "Season food with spices instead of salt." C. "Avoid the use of a straw to drink liquids." D. "Eat foods at hot temperatures."

B

A nurse is reinforcing dietary teaching with a client who has AIDS. Which of the following instructions should the nurse include in the teaching? A. Discard leftovers after 8 hours B. Use a separate cutting board for cutting poultry C. Thaw frozen foods at room temperature D. Store cold foods at 10°C (50°F) or less

PID

sepsis and death may occur if treatment is delayed or inadequate with

gonorrhea

sexually transmitted bacterial infection, can be asymptomatic

BPH

urinary retention due to bladder outlet obstruction

Kegel exercises, vaginal pessary, bowel management

3 no surgical treatments for pelvic organ prolapse

VDRL, RPR, TRUST

3 test for syphillis

Atypical hyperplasia, fibroadenoma, fibrocystic changes and cysts

3 types of benign breast disorders

Lymphocyte counts, antibody-antigen test, viral load test

3 types of lab test for HIV

Sexual contact, parenterally (IV), perinatally (mother to child)

3 ways HIV is transmitted

Hematocrit, TSH, pregnancy test, transvaginal ultrasound

4 labs tests for uterine leiomyomas

palms, soles, trunk, mucous membranes

4 places that secondary syphillis can cause rash

ANS: C HPV DNA testing can be done at the same time as the pap smear. Most women have HPV infection during their lives; however, it is not always benign. Two types, 16 and 18 are responsible for about 70% of cervical cancers. The vaccination with Gardasil 9 can be given up to age 45.

A 25-year-old woman is concerned about contracting HPV. What information by the nurse is most appropriate? a. "HPV is a benign infection that usually clears up on its own." b. "You are too old to receive the HPV vaccination." c. "We can provide HPV testing along with your Pap smear." d. "HPV is not a common sexually transmitted disease."

ANS: A, C, E With uterine leiomyomas or fibroids, heavy bleeding is the predominant symptom, with anxiety occurring because of fears of cancer or infertility. Interventions would be directed to the heavy bleeding and anxiety relief, such as relaxation techniques and education about the pathophysiology and possible treatment of the fibroids. While many women do not experience pain with this condition, some do, so the nurse would teach nonpharmacologic comfort measures. The nurse could suggest resources to give more information about the diagnosis. Discussion of the possibility of infertility and side effects of laparoscopic surgery is premature and may increase the anxiety.

A 28-year-old client is diagnosed with uterine leiomyoma and is experiencing severe symptoms. Which actions by the nurse are the most appropriate at this time? (Select all that apply.) a. Teach nonpharmacologic comfort measures. b. Discuss the high risk of infertility with this diagnosis. c. Relieve anxiety by relaxation techniques and education. d. Discuss in detail the side effects of laparoscopic surgery. e. Review complete blood count for possible iron deficiency anemia.

ANS: D Gardasil is used to provide immunity for HPV types 6, 11, 16, and 18 and Gardasil 9 protects against 5 more strains. The vaccine is recommended for people aged 9 to 26 years of age, but Gardasil 9 can be given up to age 45. Both males and females can get the vaccine. Depending on the timing and type of vaccine, either 2 to 3 doses are required.

A 30-year-old male client is asking the nurse about the vaccine for human papilloma virus (HPV). Which statement by the nurse is accurate? a. "Gardasil protects against all HPV strains." b. "You are too old to receive the vaccine." c. "Only females can receive the vaccine." d. "You will only need 1 dose of the vaccine."

C

A 32-year-old client has small uterine fibroids and is considering options for treatment. To assist the woman to make a decision about whether to have magnetic resonance-guided focused ultrasound or uterine artery embolization, what will the nurse determine? A. If there is a history of abortion in the past B. Client's risk for uterine cancer C. Whether the client desires to have children D. Age of onset of first menstrual period

C

A 48-year-old woman with the BRCA genetic mutation requests information about early detection for her daughter due to genetic risk. Which information will the nurse convey? A. Breast self-examination (BSE) beginning at 20 years of age is the best way to detect breast cancer. B. Hormone replacement therapy (HRT) combining estrogen and progesterone may be recommended by your daughter's primary health care provider. C. Cancer screening for the daughter would begin at age 38. D. The health care provider will discuss prophylactic mastectomy as the definitive option for prevention.

ANS: D The prostate-specific antigen is increased from the normal of 0 to 2.5, which could indicate benign prostatic hyperplasia or prostate cancer. Further testing would have to be done. The other values are within the normal range for males.

A 67-year-old male client had serum laboratory tests performed during his annual examination. The nurse reviews his results, as follows: testosterone: 680 ng/dL (23.6 nmol/L); prostate-specific antigen: 10 ng/mL (10 mcg/L); prolactin: 5 ng/mL (217.4 pmol). What action by the nurse is best? a. Assess for possible galactorrhea with breast discharge. b. Note the possibility of a testicular tumor. c. Communicate that results were normal. d. Prepare the client for further diagnostic testing.

C

A 68-year-old client who has had normal Pap results for 10 years and no history of cancer asks about scheduling a Pap smear. What is the appropriate nursing response? A. "You will need a Pap test this year." B. "You aren't due for a Pap test until next year." C. "You do not need to have further Pap tests at this time."* D. "You do not need a Pap tests unless you are sexually active."

ANS: B An increased size of the uterus is an abnormal finding and would be assessed further. Normal changes in the reproductive system related to aging include the graying and thinning of pubic hair, decreased size of the labia majora and clitoris, and loss of tone and elasticity of the pelvic ligaments and connective tissue. The uterus would normally be decreased, not increased, in size due to changes in hormonal levels and atrophy.

A 72-year-old woman is being assessed by the nurse for an annual physical. Which finding is of concern to the nurse? a. Thinning of pubic hair b. Increased size of the uterus c. Decreased size of the clitoris d. Loss of tone of the pelvic ligaments

B,C,D,E

A FtM client is beginning testosterone therapy. What physical change does the nurse tell the client to anticipate? (Select all that apply.) Select all that apply. A. Decreased libido B. Fat redistribution C. Increased muscle mass D. Menstruation cessation E. Deepening voice

C,D,E

A MtF client is beginning estrogen therapy. Which data obtained from the client's history are of concern to the nurse? (Select all that apply.) Select all that apply. A. History of environmental allergies B. Client has a twin sibling C. Body mass index of 32 D. One pack per day smoker E. Takes medication for blood pressure control

ANS: A, B, D, E The nurse should keep all clients at the office for at least 30 minutes after the administration of benzathine penicillin G. Allergic signs and symptoms consist of rash, shortness of breath, chest tightness, and anxiety. Heart irregularity and confusion are not seen as an allergic manifestation.

A client being treated for syphilis visits the office with a possible allergic reaction to benzathine penicillin G. Which abnormal findings would the nurse expect to document? (Select all that apply.) a. Red rash b. Shortness of breath c. Heart irregularity d. Chest tightness e. Anxiety f. Confusion

ANS: A, B, C, E Organic erectile dysfunction can be caused by surgical procedures, vascular diseases such as hypertension and its treatment, diabetes mellitus, and alcohol consumption. There is no evidence that exercise or hot baths are related to this problem.

A client came to the clinic with erectile dysfunction. What are some possible causes of this condition that the nurse could discuss with the client during history taking? (Select all that apply.) a. Recent prostatectomy b. Long-term hypertension c. Diabetes mellitus d. Hour-long exercise sessions e. Consumption of beer each night f. Taking long hot baths

ANS: A These are symptoms of possible testicular cancer. AFP is a tumor marker that is elevated in testicular cancer. PSA and PAP testing is used in testing for prostate cancer and its metastasis. CRP is diagnostic for inflammatory conditions.

A client comes to the clinic with concerns about an enlarged left testicle and heaviness in his lower abdomen. Which diagnostic test would the nurse expect to be ordered? a. Alpha-fetoprotein (AFP) b. Prostate-specific antigen (PSA) c. Serum acid phosphatase (PAP) d. C-reactive protein (CRP)

D

A client had a total abdominal hysterectomy 2 days ago and is to be discharged on antibiotics. What does the nurse include in discharge teaching? A. "Stop the antibiotic if you feel nauseated because it will lose its effectiveness." B. "You will need to take the drug until your incision heals." C. "After your first day at home, you can stop them if you do not have a fever." D. "It is important to take them as directed until they are all gone."

ANS: B Epidural anesthesia will cause the client to not be able to move (or feel) the legs for several hours. It is important for client safety that adequate help is available prior to this client trying to get out of bed. Securing the catheter to the leg and monitoring dressings and drainage are important for any client after surgery. Positioning the drain to the contralateral side is not needed.

A client had a vaginoplasty under epidural anesthetic. Which action by the nurse is most important? a. Ensure that the urinary catheter is securely attached to the leg. b. Instruct the client not to try to get out of bed unassisted. c. Monitor the patient's dressings and wound drainage. d. Position the Jackson-Pratt drain to the contralateral side.

B

A client has a positive HSV-2 test but is asymptomatic. What action by the nurse is best? a. Encourage the client to have frequent STI screening. b. Teach the client ways to prevent getting STIs. c. Provide the same education as if the client were symptomatic. d. Inform the client that partner notification is unnecessary.

C

A client has been scheduled for a transvaginal ultrasound. Which allergy does the nurse identify that should be immediately reported to the healthcare provider? A. Eggs B. Corn C. Latex D. Iodine

ANS: C This client has signs of a Jarisch-Herxheimer reaction which is caused when the organisms' cell walls are disrupted and cellular contents are released rapidly. It is usually self-limiting and benign. Antipyretics and mild analgesics treat the symptoms. The client does not need to monitor for shortness of breath, come in to the clinic, or get antihistamines for an allergic reaction.

A client has been treated for syphilis with IM penicillin. The next day the client calls the clinic to report fever, chills, achy muscles, and a worsening rash. What statement by the nurse is most appropriate? a. "You must be allergic to penicillin; over the counter antihistamines will help." b. "Please go to the nearest emergency department if you develop shortness of breath." c. "You can take acetaminophen or ibuprofen for the pain and achiness." d. "I think you should come in to the clinic either today or tomorrow and be checked."

ANS: C Health care professionals need to avoid the arm on the side of the surgery for blood pressure measurement, injections, or blood draws. Since lymph nodes are removed, lymph drainage would be compromised. The pressure from the blood pressure cuff could promote swelling. Infection could occur with injections and blood draws. Checking urine output, elevation of the affected arm on a pillow, and encouraging beginning exercises are all safe postoperative interventions.

A client has just returned from a right radical mastectomy. Which action by the assistive personnel (AP) would require the nurse consider to intervene? a. Checking the amount of urine in the catheter collection bag b. Elevating the right arm on a pillow c. Taking the blood pressure on the right arm d. Encouraging the client to squeeze a rolled washcloth

ANS: A, B, C, D, E Possible complications of PID include chronic pelvic pain, infertility, ectopic pregnancy, tubo-ovarian abscess, peri-hepatitis, inflammation of the liver capsule, and inflammation of the peritoneal surfaces of the anterior right upper quadrant.

A client has pelvic inflammatory disease (PID). What complications does the nurse monitor the client for? (Select all that apply.) a. Chronic pelvic pain b. Infertility c. Ectopic pregnancy d. Tubo-ovarian abscess e. Peri-hepatitis f. Pancreatitis

ANS: B Type II endometrial cancer is likely to invade the uterine wall and metastasize. Treatment with brachytherapy is intended to prevent recurrence. During the treatment, which lasts 20 to 30 minutes each, the woman must remain on bedrest to avoid dislodging the radioactive source. The source emits radiation while it is in place, so the woman is in the treatment room by herself. Once it is removed, she has no restrictions on being around others. She would need to report any heavy vaginal bleeding or severe diarrhea.

A client has recently been diagnosed with type II endometrial cancer and will be treated with brachytherapy. What statement by the client indicates a need for further education on this treatment? a. "Each treatment will take only 20 to 30 minutes." b. "I have to be alone in the room during treatment so I don't expose others." c. "I can get up and walk around or read in a chair during the treatments." d. "I need to report any heavy vaginal bleeding or severe diarrhea.

D

A client has recently had a vaginoplasty and has noticed stool coming from her vagina. The telehealth nurse advises her to immediately go the emergency department. What is the nurse's concern? A. Prolapse B. Labial hematoma C. Dislodgement of the urinary catheter D. Vaginal-rectal fistula

ANS: A, B, E Clients should avoid hot water baths as they may increase the itching and infection. They may take warm or tepid sitz baths for 30 minutes several times a day to relieve itching. Clients should cleanse the inner labia mucosa with water, not soap, during a bath or shower. All of the other options are correct.

A client has recurrent vulvovaginitis. Which statements by the client indicate a need for further teaching? (Select all that apply.) a. "I can take a long, hot bath to relieve itching." b. "I need to take all of my antibiotics as prescribed." c. "I should avoid having sex until my infection is gone." d. "I should not douche or use feminine hygiene sprays." e. "I should use antibacterial soap to clean the area." f. "I should switch to wearing only cotton underwear."

ANS: B Bright red urinary drainage with clots may indicate arterial bleeding. The nurse would notify the primary health care provider immediately and begin irritating the catheter with sterile normal saline (not sterile water). The nurse can delegate the vital signs. The nurse would review hemoglobin and hematocrit and would remind the client not to pull on the catheter for all clients with bladder irrigation. But for this client who may be bleeding the nurse would take further action to address the problem.

A client has returned from a transurethral resection of the prostate with a continuous bladder irrigation. Five hours after the operation, the nurse notes the drainage is bright red with clots. What action should the nurse take first? a. Review the most recent hemoglobin and hematocrit. b. Take vital signs and begin immediate irrigation with sterile water. c. Notify the primary health care provider immediately. d. Remind the client not to pull on the catheter.

ANS: A Ice is applied to the perineum intermittently to reduce bruising, pain, and discomfort. Elevating the legs on pillows is not recommended after a lengthy procedure in the lithotomy position, which predisposes the client to venous thromboembolism. Positioning the client on the left side and raising the head of the bed are not comfort measures related to this procedure.

A client has returned from the postanesthesia care unit after a vaginoplasty. What comfort measure does the nurse provide for this client? a. Apply ice to the perineum. b. Elevate the legs on pillows. c. Position the client on the left side. d. Raise the head of the bed.

D

A client has undergone a prostate biopsy. Which postprocedure symptom will the nurse teach the client to report immediately to the primary health care provider? A. Discoloration of the semen 5 days after biopsy B. Light rectal bleeding 2 days after procedure C. Tenderness at the site 1 day after biopsy D. Pain upon urination 3 days after procedure

ANS: D Use of vaginal estrogen cream and gentle dilation can help with vaginal changes and loss of libido. Weight gain and masculinization are misperceptions after a vaginal hysterectomy. Vitamins, supplements, a balanced diet, and exercise are helpful for healthy living, but are not necessarily going to increase libido.

A client has undergone a vaginal hysterectomy with a bilateral salpingo-oophorectomy. She is concerned about a loss of libido. What intervention by the nurse would be best? a. Suggest increasing vitamins and supplements daily. b. Discuss the value of a balanced diet and exercise. c. Reinforce that weight gain may be inevitable. d. Teach that estrogen cream inserted vaginally may help.

B, D

A client has undergone transurethral resection of the prostate (TURP). Which intervention does the nurse incorporate in the postoperative plan of care? (Select all that apply.) Select all that apply. A. Perform intermittent urinary catheterization every 4 to 6 hours. B. Assist to mobilize as soon as permitted. C. Encourage urination around the catheter if pressure is felt. D. Administer antispasmodic medications. E. Place in a supine position with his knees flexed.

ANS: B The source of infection should be removed first. All of the other answers are possible interventions depending on the client's symptoms and vital signs, but removing the tampon is the priority

A client is admitted to the emergency department with toxic shock syndrome. Which action by the nurse is the most important? a. Administer IV fluids to maintain fluid and electrolyte balance. b. Remove the tampon as the source of infection. c. Collect a blood specimen for culture and sensitivity. d. Transfuse the client to manage low blood count.

D

A client is beginning transdermal estrogen (Climara) therapy. Which client statement requires additional health teaching by the nurse? A. "This medicine will increase my risk of blood clots." B. "I will monitor my blood pressure while I am taking this medicine." C. "My blood work will be checked regularly." D. "I will need to change out the patch once a month."

D

A client is being discharged after a total abdominal hysterectomy (TAH). What principle guides the nurse in planning discharge care and teaching? A. Clients in their childbearing years generally adapt better. B. Psychological reactions would be evident by discharge. C. The client can resume normal activities upon discharge. D. No special home equipment will be necessary for the client.

ANS: B There needs to be a certain level of body fat and weight to maintain regular menstrual cycles. The client has only indicated that she has increased her workouts. There is no indication that she has anorexic or bingeing and purging behaviors. The question about wanting to talk about needing that much exercise sounds judgmental.

A client is concerned about her irregular menstrual periods since she has increased her daily workouts at the gym to 2 hours each day. What is the nurses' best response? a. "Do you want to talk about the need for that much exercise?" b. "Exercise is healthy but can decrease body fat and cause irregular periods." c. "Bingeing and purging can cause electrolyte problems in your body." d. "Anorexic behavior can result in decreased estrogen levels."

ANS: B Injury and infection are risk factors for lymphedema; therefore, the client needs to be cautious with activities using the affected arm. Radiation therapy is just one of the factors that could cause lymphedema. Other risk factors include obesity and presence of axillary disease. The symptoms of lymphedema are heaviness, aching, fatigue, numbness, tingling, and swelling, and are not common after the surgery. Women with lymphedema live fulfilling lives.

A client is concerned about the risk of lymphedema after a mastectomy. Which response by the nurse is best? a. "You do not need to worry about lymphedema since you did not have radiation therapy." b. "Be careful not to injure that arm or get any infection in that arm." c. "Numbness, tingling, and swelling are common sensations after a mastectomy." d. "The risk for lymphedema is a real threat and can be very self-limiting."

ANS: D Frequent visits to the bathroom during the night could cause sleep interruptions and affect the client's mood and mental status. Telling the client his symptoms should not lead to less socialization is patronizing. Instructing the client to seek more pleasant things to do also is patronizing. Neither statement has any information the client could find useful. The statement about age has no validity and again does not offer useful information.

A client is diagnosed with benign prostatic hyperplasia and seems sad and irritable. After assessing the client's behavior, which statement by the nurse would be the most appropriate? a. "The urine incontinence should not prevent you from socializing." b. "You seem depressed and should seek more pleasant things to do." c. "It is common for men at your age to have changes in mood." d. "Nocturia could cause interruption of your sleep and cause changes in mood."

ANS: A The drainage tubes (such as a Jackson-Pratt drain) lie just under the skin but need to be removed by the health care professional in about 1 to 3 weeks at an office visit. Drainage should be less than 30 mL for three consecutive days. The client should be aware of her positioning to prevent kinking of the tubing. A foul odor from the drainage may indicate an infection; the primary health care would be contacted immediately.

A client is discharged to home after a modified radical mastectomy with two drainage tubes. Which statement by the client would indicate that further teaching is needed? a. "I am glad that these tubes will fall out at home when I finally shower." b. "I should measure the drainage each day to make sure it is less than an ounce (30 mL)." c. "I should be careful how I lie in bed so that I will not kink the tubing." d. "If there is a foul odor from the drainage, I will contact my primary health care provider."

ANS: A During the procedure, the client may experience light-headedness, so the nurse would assist her with sitting up afterwards for safety. The nurse does provide a pad to prevent any staining from the dye and does inform the client of the possibility of shoulder pain, but an action to prevent injury is more important. Antibiotics are not prescribed afterward.

A client is having a hysterosalpingogram. What action by the nurse is most important? a. Assist the client in sitting up after the procedure. b. Provide the client with a pad to avoid dye stains on the clothes. c. Teach her to take all antibiotics prescribed until finished. d. Inform the client that the procedure may cause shoulder pain.

ANS: A, D, E, F Risk factors for prostate cancer include having a first-degree relative with the disease, advanced age, and African-American race. Smoking, obesity, and eating too much red meat are not considered risk factors. Research is exploring the relationship with diet.

A client is interested in learning about the risk factors for prostate cancer. Which factors does the nurse include in the teaching? (Select all that apply.) a. First-degree relative with prostate cancer b. Smoking c. Obesity d. Advanced age e. Eating too much red meat f. Race

ANS: A Doxorubicin can cause cardiotoxicity with symptoms of extreme fatigue, shortness of breath, chronic cough, and edema. These need to be reported as soon as possible to the primary health care provider. Nausea, vomiting, hair loss, and mucositis are common problems associated with chemotherapy regimens.

A client is placed on a medical regimen of doxorubicin (Adriamycin), cyclophosphamide, and fluorouracil for breast cancer. Which side effect seen in the client would the nurse report to the primary health care provider immediately? a. Shortness of breath b. Nausea and vomiting c. Hair loss d. Mucositis

ANS: B Male-to-female clients can consult with a speech-language pathologist for vocal training to help with intonation and pitch. While vocal surgery is possible, it may not be the best first option due to cost and invasiveness. Telling the client that there will be no change to the voice does not give the client information to address the concern. While the hormones this client is taking will not affect the voice, simply stating that fact does not help the client manage this issue.

A client is preparing for MtF gender-affirming surgery. The client is worried about the voice not sounding feminine enough. What action by the nurse is best? a. Ask if the client has considered vocal cord surgery to change the voice. b. Refer the client for vocal therapy with a speech-language pathologist. c. Teach the client that there will be no effect on the patient's voice. d. Tell the client that the use of hormones will eventually change the voice.

ANS: D The client is taught that she should not participate in strenuous activity for a week after the procedure. Carbon dioxide is infused into the pelvic cavity to visualize the organs. There are only one or more small incisions with this procedure. The referred shoulder pain that will occur only lasts 48 hours.

A client is scheduled for a laparoscopy to remove endometriosis tissue. Which response by the client alerts the nurse of the need for further teaching? a. "The surgeon told me that carbon dioxide would be infused into my pelvic cavity." b. "There will be one or more small incisions in order to visualize all of the organs." c. "There will be some shoulder pain after the procedure that may last 48 hours." d. "I can return to jogging my 3-mile (5 km) routine in a few days."

A

A client is scheduled for a total hysterectomy with a laparoscopic vaginal approach after a diagnosis of microinvasive cervical cancer. What psychological and/or social changes does the nurse expect this client to experience? A. Sexual counseling may be needed, especially if the client has doubts about her ability to feel like a woman and engage in sexual activities. B. The client will be actively involved in her own care in the immediate postoperative period. C. Because the surgery does not affect a visible site, altered body image issues are not as common. D. The client would demonstrate reality testing and would experience a grief reaction immediately after her surgery.

ANS: B, C, D, F EBRT is usually performed in ambulatory care and does not require hospitalization. The client needs to know to evaluate the skin, especially in the perineal area, for any breakdown, and avoid sunbathing. The technician does not apply new site markings, so the client needs to avoid washing off the markings that indicate the treatment site.

A client is scheduled to start external beam radiation therapy (EBRT) for her endometrial cancer. Which teaching by the nurse is accurate? (Select all that apply.) a. "You will need to be hospitalized during this therapy." b. "Your skin needs to be inspected daily for any breakdown." c. "It is not wise to stay out in the sun for long periods of time." d. "The perineal area may become damaged with the radiation." e. "The technician applies new site markings before each treatment." f. "You will not be radioactive or pose any danger to anyone else."

ANS: C Tamoxifen reduces the estrogen available to breast tumors to stop or prevent growth. This drug does not block the release of luteinizing hormone to prevent the ovaries from producing estrogen; leuprolide does this. Chemotherapy agents interfere with cancer cell division. Newer research supports treatment with tamoxifen for 10 years to prevent recurrence.

A client is starting hormonal therapy with tamoxifen to lower the risk for breast cancer. What information needs to be explained by the nurse regarding the action of this drug? a. It blocks the release of luteinizing hormone. b. It interferes with cancer cell division. c. It selectively blocks estrogen in the breast. d. It inhibits DNA synthesis in rapidly dividing cells.

B

A client is struggling with body image after breast cancer surgery. Which behavior indicates to the nurse that the client's coping is maladaptive? A. Requesting a temporary prosthesis B. Avoiding eye contact with staff C. Saying, "I feel like less of a woman" D. Saying, "This is the ugliest scar ever"

ANS: C This client has signs and symptoms of testicular torsion, which is a surgical emergency. For client safety, the nurse assesses last oral intake. Rating the pain is an important intervention too but is not related to safety. The client cannot have opioids prior to signing a surgical consent. The client does not have signs and symptoms of testicular cancer.

A client presents to the emergency department reporting vomiting, severe lower abdominal pain, and a tender mass above one testis. What action by the nurse is most important? a. Have the client rate pain using the 0-10 scale. b. Prepare to administer an IV opioid analgesic. c. Determine when he last ate or drank anything. d. Assess risk factors for testicular cancer.

C

A client reports taking a supplement that his partner recommended for benign prostatic hyperplasia (BPH). Which supplement does the nurse anticipate the client is taking? A. Magnesium B. Calcium C. Saw palmetto D. Feverfew

ANS: C Headache, dizziness, and shortness of breath are symptoms of possible TURP syndrome in which the irrigation fluid is absorbed, putting strain on the client's heart. The nurse notifies the primary health care provider immediately as the client may need intensive care monitoring. There is no data indicating the client needs a blood transfusion, plus that would add even more fluid in the system. The irrigant may need to be slowed but that is not the first action the nurse would take. Vital signs do need to be taken frequently in this situation, but the nurse notifies the primary health care provider first.

A client returned from a transurethral resection of the prostate 8 hours ago with a continuous bladder irrigation. The client reports headache and dizziness. What action by the nurse is most appropriate? a. Consider starting a blood transfusion. b. Slow the bladder irrigation down. c. Report the findings to the surgeon immediately. d. Take the vital signs every 15 minutes.

B

A client scheduled for a hysterosalpingogram is interviewed by the nurse. What interview information is critical for the nurse to report to the primary health care provider before the procedure? A. Administration of a rectal suppository 4 hours ago B. Allergy to shellfish C. Menstrual period that ended 3 days ago D. Abortion 2 months ago

D

A client tells the nurse in the gynecology clinic that she doesn't get a yearly mammogram because she is afraid of what might be found. Which teaching will the nurse provide? A. People with low breast cancer risk can obtain an MRI instead. B. Detection of breast cancer before or after axillary node invasion yields the same outcome. C. Mammography is needed only if the client has a first-degree relative with breast cancer. D. Early detection is important, as localized breast cancer has a 99% 5-year survival rate

A

A client who had an anterior colporrhaphy is being discharged. What does the nurse tell the client before her discharge? A. "Use a pillow to splint your abdomen." B. "Avoid lifting more than 25 lb (11.3 kg)." C. "Return to the clinic in 6 weeks for suture removal." D. "Do not have sexual intercourse for at least 2 weeks."

A

A client who has been diagnosed with breast cancer tells the nurse she wishes to use only natural and complementary interventions. What teaching will the nurse provide? A. This type of therapy would not replace standard treatment. B. If chemotherapy has been recommended, complementary therapies are contraindicated. C. Complementary therapies can only be used after surgery. D. There are many natural herbs that have been shown to treat cancer.

C

A client who has recently been prescribed finasteride reports dizziness, cold sweats, and chills. What will the nurse tell this client? A. "You will need to see your primary health care provider." B. "You need to stop taking the medication immediately." C. "These are common effects that usually diminish over time." D. "These effects are not related to the finasteride."

D

A client who has undergone breast surgery is struggling her sexuality. How will the nurse address the client's concerns? A. Remind the client the she needs to avoid sexual intercourse at this time. B. Ask the client if she is using her surgery as an excuse not to avoid intercourse. C. Give the client a business card for a local counselor. D. Ask the client about satisfaction with sexual relations with her partner.

A

A client who is scheduled for a Pap smear reports having sexual intercourse this morning and douching afterward. What is the appropriate nursing action? A. Reschedule the Pap smear for another week B. Delay the procedure until later in the afternoon C. Help the client prepare for the procedure at this time D. Hold the procedure until the client's next menstrual cycle

B

A client with a history of BPH calls the telehealth nurse reporting the sudden onset of testicular pain after moving heavy furniture. What is the appropriate nursing response? A. "Taking ibuprofen may help alleviate the pain." B. "Please go to your closest emergency department right away." C. "This is a common reaction when performing labor; the pain will go away." D. "Your BPH is probably giving you difficulty because you were moving furniture."

A, C, E

A client with benign prostatic hyperplasia is being discharged with a prescription for prazosin. Which teaching will the nurse include? (Select all that apply.) Select all that apply. A. Avoid drugs used to treat erection problems. B. Take the medication in the afternoon. C. Keep all appointments for follow-up laboratory testing. D. Hearing tests will need to be conducted periodically. E. Be careful when changing positions.

ANS: C A client with genital herpes can still spread the disease when asymptomatic through viral shedding. The client is taught to use condoms with all sexual activity. Pouring water over the genitals (or urinating in the shower) will help decrease the pain of urine passing over open lesions. Good handwashing is important. Open lesions can lead to fluid loss so the client is taught to increase fluid intake.

A client with genital herpes has painful blisters on her vulva. After teaching the client self-care measures, which statement indicates the need for further education? a. "Pouring water over my genitals will decrease the pain of urinating." b. "I will wash my hands carefully after applying ointment." c. "When I don't have lesions, I am not contagious to my sexual partner." d. "I should increase my fluid intake when I have open lesions."

ANS: B Flutamide is an antiandrogen drug that can cause liver toxicity. The nurse would follow up on the statement that the client's eyes may be looking a little yellow which could indicate the onset of this adverse effect. Leuprolide can cause osteoporosis, hot flashes, and gynecomastia. The statements regarding weight-bearing exercise, enlarging breasts, and hot flashes are not cause for concern.

A client with metastatic prostate cancer has been prescribed leuprolide, a bisphosphonate, and flutamide. Which statement by the client warrants further investigation by the nurse? a. "I go for a short walk each day, even when I am very tired." b. "My wife has noticed my eyes looking a little yellow." c. "I ordered some looser shirts to hide my enlarging breasts." d. "Now I understand my wife's hot flashes with menopause."

ANS: A This client has signs of gonorrhea. Co-infection with gonorrhea and chlamydia is common, so the client being treated for gonorrhea also needs treatment for chlamydia with oral antibiotics. It is fairly accurate to say two medications increases the chance of cure, but does not really explain the situation. Giving the client two medications is not because some people are not affected by the injection nor is it to prevent needing a 3-month follow-up test. Testing for re-infection with chlamydia is recommended by the CDC.

A client with multiple sexual partners has been assessed for symptoms of dysuria and green, malodorous vaginal discharge. The nurse administers and injection of ceftriaxone and gives the client a prescription for doxycycline. The client asks why two drugs are needed. What answer by the nurse is best? a. "It is very common to be infected with both gonorrhea and chlamydia." b. "Giving two medications increases the chance of curing the infection." c. "Some people are not affected by the injection and need more medication." d. "This will prevent you from needing a 3-month follow-up test."

A

A client with newly diagnosed gynecologic cancer is being discharged home. Which health care team member does the nurse contact to coordinate nursing care at home for this client? A. Case manager B. Hospice C. Social services D. Primary health care provider

D

A client with newly diagnosed pelvic inflammatory disease (PID) is prescribed an antibiotics as an outpatient. What teaching does the nurse provide about the treatment regimen? A. "If you engage in sexual activity, be certain to use a latex condom." B. "A rise in your temperature is expected for the first several days." C. "Return to the clinic in 7 to 10 days after completing treatment for a checkup." D. "Rest in a semi-Fowler position to help with the infection and pain."

ANS: D Chills and fever could indicate a persistent infection and the immediate need to alter the dose or type of antibiotic. Anger is a normal reaction to a sexually transmitted infection and the pain of pelvic inflammatory disease. Gastrointestinal symptoms are common side effects of antibiotics but not an immediate cause for intervention.

A client with pelvic inflammatory disease is seen by the nurse 72 hours after starting oral antibiotics. Which finding leads the nurse to take immediate action? a. Feelings of anger that her partner infected her b. Loose stools over the last 2 days c. Anorexia and nausea d. Chills and a temperature of 101° F (38.3° C)

A

A client with pelvic pain is admitted to the same-day surgery unit for a laparoscopic procedure. Which nursing action will the RN delegate to assistive personnel (AP)? A. Taking admission blood pressure and heart rate B. Educating about analgesic use for referred pain C. Teaching about postoperative activity restrictions D. Inserting a retention catheter using sterile technique

ANS: B Gardasil and Gardasil 9 are used to provide immunity for HPV types 6, 11, 16, and 18 and others that are high risk for cervical cancer and genital warts. While there is some truth that urination after intercourse may decrease the risk of infection by flushing out organisms, it does not eliminate the risk of contaminating bacteria traveling up the urethra or from skin-to-skin contact. The other statements are not accurate.

A college student seeks information from the school's nurse about how to avoid sexually transmitted infections (STIs) without abstinence as a choice. Which statement by the nurse is best? a. "Urinating after intercourse will eliminate the risk of infection." b. "A vaccine can prevent genital warts caused by some strains of the human papilloma virus (HPV)." c. "Oral contraception can prevent pregnancy and STIs." d. "Good handwashing helps prevent infection associated with STIs."

ANS: C There is a good possibility that the boyfriend reinfected the client after the medication regimen was finished. Both the client and the boyfriend need to be treated. The other statements were in compliance with the recommendations of abstinence and the usual medication regimen with doxycycline. Wine should not interfere with the treatment.

A female client returned to the clinic with a yellow vaginal discharge after being treated for Chlamydia infection 3 weeks ago. Which statement by the client alerts the nurse that there may be a recurrence of the infection? a. "I did practice abstinence while taking the medication." b. "I took doxycycline two times a day for a week." c. "I never told my boyfriend about the infection." d. "I did drink wine when taking the medication for Chlamydia."

C

A female-to-male client wishes to retain the option of having biologic children after transitioning. What available option does the nurse suggest to the client? A. Oocyte freezing can occur after hormone therapy has started and before menstruation ceases. B. Oocyte freezing can occur any time before gender reassignment surgery. C. Oocyte freezing can occur prior to hormone therapy or gender reassignment surgery. D. No options are available to a client with gender dysphoria.

ANS: B Benzathine penicillin G is the evidence-based treatment for primary, secondary, and early latent syphilis. The client needs to be assessed for allergies before treatment. The other questions would be helpful in the client's history of sexually transmitted infections but not as important as knowing whether the client is allergic to penicillin.

A male client is diagnosed with primary syphilis. Which question by the nurse is a priority at this time? a. "Have you been using latex condoms?" b. "Are you allergic to penicillin?" c. "When was your last sexual encounter?" d. "Do you have a history of sexually transmitted infections?"

C

A male-to-female transgender client wishes to discuss breast augmentation surgery. What client statement indicates the need for further education by the nurse? A. "My insurance may not pay for this surgery." B. "If I take hormones, I will have to get a mammogram." C. "Fat injections can be used instead of implants for a more natural look." D. "I may need to take hormones for about a year before my surgery."

ANS: B After a radical prostatectomy, the nurse would not provide a rectal suppository for constipation. All rectal treatments are contraindicated. The nurse would delegate emptying and recording drainage, remove the sequential pressure devices when clients begin ambulating, and discuss long-term complications of the operation.

A new nurse care for several client after radical prostatectomies for prostate cancer. What action by the nurse indicates a need to review care measures for this type of client? a. Delegates emptying and recording contents of the drainage devices. b. Administers a suppository to the client who reports constipation. c. Removes the sequential compression stockings on ambulatory clients. d. Discusses long-term complications such as erectile dysfunction.

ANS: A Mesh is not used in the transvaginal approach as it has been discontinued in this country. The other statements show good understanding.

A nurse has taken an informed consent to a woman who is having a transvaginal repair of a prolapsed uterus. What client statement indicates a need for more information? a. "The mesh they use may become infected." b. "I may still need to do my Kegel exercises." c. "I will watch for any signs of infection." d. "I know how to use the incentive spirometer."

ANS: C Modifiable risk factors can help prevent breast cancer. The client should lessen alcohol intake and not have wine 7 days a week. Breast-feeding, regular exercise, maintaining a normal weight, and avoiding hormone replacement are also strategies for breast cancer prevention.

A nurse has taught a female client about the modifiable risk factors for breast cancer. Which statement made by the client indicates that more teaching is needed? a. "I am fortunate that I breast-fed each of my three children for 12 months." b. "It looks as though I need to start working out at the gym more often." c. "I am glad that we can still have wine with every evening meal." d. "When I have menopausal symptoms, I must avoid hormone replacement therapy."

ANS: C The client is displaying symptoms similar to secondary syphilis, with flulike symptoms and rash due to the spirochetes circulating throughout the bloodstream. Therefore, the nurse needs to further assess the client's lesions with gloves since the client is highly contagious at this stage. Tertiary syphilis may display in the form of cardiovascular or central nervous system symptoms. Neurosyphilis can appear at any time, in any state, and can include hearing loss.

A nurse is assessing a client who presents with a scaly rash over the palms and soles of the feet and the feeling of muscle aches and malaise. Which action by the nurse is most appropriate? a. Reassure the client that these lesions are not infectious. b. Assess the client for hearing loss and generalized weakness. c. Don gloves and further assess the client's lesions. d. Take a history regarding any cardiovascular symptoms.

A

A nurse is assisting with planning an educational program for a group of young adults about interventions that reduce the risk of cervical cancer. Which of the following interventions should the nurse include? A. Get the human papillomavirus (HPV) immunization B. Avoid the use of tampons on a routine basis C. Avoid drinking alcohol D. Get a Papanicolaou test every year starting at age 30

C

A nurse is caring for a client who tests positive for human immunodeficiency virus (HIV). The client asks the nurse, "Should I tell my partner that I am HIV positive?" Which of the following is an appropriate nursing response? A. "That is your decision alone." B. "I would if I were you." C. "It sounds like you are unsure what to say to your partner." D. "Your provider is required by law to notify your partner."

B

A nurse is caring for a male client who reports a thick urethral discharge. Which of the following actions should the nurse take? A. Contact the client's sexual partners B. Obtain a urethral specimen for culture C. Prepare to administer penicillin to the client D. Obtain blood for a rapid plasma reagin test

ANS: B The fluid that is used during this procedure to distend the uterine cavity can be absorbed, leading to fluid overload. This client has signs of fluid overload which can be critical. The nurse would notify the Rapid Response Team first, then perform the other actions.

A nurse is caring for a woman who had hysteroscopic surgery for uterine leiomyomas. On initial assessment, the nurse notes the following: pulse: 114 beats/min, respiratory rate: 20 breaths/minute, crackles in bilateral lung bases. What action by the nurse takes priority? a. Assess the client for pain. b. Call the Rapid Response Team. c. Obtain an oxygen saturation. d. Delegate a temperature.

D

A nurse is collecting data from a client who has Kaposi sarcoma. Which of the following findings should the nurse expect with this condition? A. Nonproductive cough with a fever and shortness of breath B. Lesions on the retina that produce blurred vision C. Onset of progressive dementia D. Reddish-purple skin lesions

C

A nurse is collecting data from a client who has an exacerbation of herpes zoster. Which of the following manifestations of the client's skin should the nurse expect to see? A. Confluent, honey-colored, crusted lesions B. Large tender nodule located on a hair follicle C. Unilateral, localized, nodular skin lesions D. Fluid-filled vesicular rash in the genital region Next Question Feedback

ANS: A, B, C, E Gender dysphoria is emotional distress caused by the incongruence between natal sex (sex assigned at birth) and gender identity. Gender identity is a person's inner sense of being a male, a female, or an alternative gender Natal sex describes the gender a person is born with or is assigned to at birth. Transgender is an adjective that describes individuals who self-identify as the opposite gender or a gender that does not match their natal sex. A trans-woman is a natal male who identifies and/or lives as a woman.

A nurse is learning about the health care needs of individuals who identify as LGBTQIA+ and transgender. Which terms are correctly defined? (Select all that apply.) a. Gender dysphoria—distress caused by incongruence between natal sex and gender identity. b. Gender identity—a person's inner sense of being a male, a female, or an alternative gender. c. Natal sex—the sex one is born with or is assigned to at birth. d. Transgender—a person who dresses in the clothing of the opposite sex. e. Trans-woman—a male who identified or lives as a woman.

ANS: C Clients in certain high risk groups should discuss screening for prostate cancer with their primary health care providers at age 45. High risk groups include African Americans and men with a first-degree relative who was diagnosed with prostate cancer before the age of 65. This new client will be encouraged to discuss screening even though he is past the age of initial discussion. Recommendations do vary somewhat, but he is in a recognized high risk group. The nurse would not say that he "should" be screened annually. Screening is not recommended for men over the age of 70.

A nurse is providing education to a new 55-year-old African-American client about screening for prostate cancer. What action by the nurse is most appropriate? a. Inform the client that recommendations vary, so screening is a personal choice. b. Let the client know that as an African American, he should be screened annually. c. Teach the client that he is in a high risk group and should discuss screening. d. Give the client written information that discourages screening until age 70.

ANS: A The MtF client retains the prostate, so annual screening examinations for prostate cancer remain important. The other statements are good general health teaching ideas for any patient

A nurse is providing health teaching to a middle-age male-to-female (MtF) client who has undergone gender-reaffirming surgery. What information is most important to this patient? a. "Be sure to have an annual prostate examination." b. "Continue your normal health screenings." c. "Try to avoid being around people who are ill." d. "You should have an annual flu vaccination.

B

A nurse is reinforcing discharge teaching with a client who has AIDS. Which of the following statements should the nurse include? A. "You can share razors with your partner as long as they are disposable." B. "You should clean bathroom surfaces with a bleach and water solution." C. "You should increase your intake of raw fruits and vegetables." D. "You can continue gardening as long as you wear a mask."

B

A nurse is reinforcing discharge teaching with the partner of a client who has acquired immune deficiency syndrome (AIDS). Which of the following statements by the client's partner indicates the need for further teaching? A. "I will dispose of soiled tissues in separate plastic bags." B. "I'll clean blood spills immediately with hot water." C. "I know that handwashing is an important preventive measure." D. "I will wash soiled clothes in hot water."

D

A nurse is reinforcing teaching with a client about preventing the transmission of hepatitis A. The nurse should identify that hepatitis A is transmitted by which of the following routes? A. Maternal-fetal B. Fecal-oral contamination C. Genital sexual contact D. Blood-to-blood

A

A nurse is reinforcing teaching with a client who has AIDS about preventing infection while at home. Which of the following instructions should the nurse include in the teaching? A. Wash genitalia using an antimicrobial soap B. Rinse dishes with cold water C. Clean toothbrush once per month D. Incorporate raw fruits and vegetables into the diet

C

A nurse is reinforcing teaching with a client who has AIDS about the transmission of Pneumocystis jiroveci pneumonia (PCP). Which of the following pieces of information should the nurse include in the teaching? A. "PCP is sexually transmitted from person to person." B. "You were most likely exposed to a contaminated surface such as a drinking glass." C. "PCP results from an impaired immune system." D. "You might have contracted PCP from a family pet."

C

A nurse is reinforcing teaching with a client who has a diagnosis of hepatitis A. Which of the following statements by the client indicates an understanding of the teaching? A. "I am unable to donate blood." B. "I will need to get a booster shot of immune serum globulin every year." C. "I should stop eating raw clams." D. "I can get this disease by getting a tattoo."

D

A nurse is reinforcing teaching with a client who has genital herpes about self-management. Which of the following instructions should the nurse include in the teaching? A. Use an alcohol-based soap to clean lesions B. Wear a condom during sexual activity when lesions are present C. Take a sitz bath once per day D. Pour running water over the lesions when urinating

B

A nurse is reinforcing teaching with a client who has human immunodeficiency virus (HIV) about how the virus is transmitted. Which of the following statements should the nurse include the teaching? A. "HIV can be transmitted as soon as a person develops manifestations." B. "HIV can be transmitted to anyone who has had contact with infected blood." C. "HIV is transmitted through the respiratory route by droplets." D. "HIV is transmitted only during the active phase of the virus."

A

A nurse is reinforcing teaching with a client who is being treated for genital warts. Which of the following statements indicates that the client understands how to prevent transmission of his sexually transmitted infection (STI)? A. "I will bring in my sexual partner for treatment." B. "Now that I've had my first dose of medicine, I can resume sexual activity." C. "After I have been treated, I don't have to use condoms." D. "Once treatment is complete and I am free of symptoms, I don't have to return to the clinic."

C

A young adult with testicular cancer is admitted for unilateral orchiectomy and retroperitoneal lymph node dissection. Which task will the nurse delegate to assistive personnel (AP)? A, Discuss reproductive options with the client and significant other. B. Evaluate the client's understanding of chemotherapy and radiation treatment. C. Encourage the client to cough and deep-breathe after surgery. D. Teach about the availability of a gel-filled silicone testicular prosthesis.

ANS: C The nurse may encounter transgender clients whose outward appearance does not match their demographic data. In this case, the nurse should greet the client and ask the client to explain his or her preferred forms of address. Lengthy apologies can often create embarrassment. The nurse should not assume that the client is not present in the room. The chart may or may not contain errors, but that is not related to determining how the client prefers to be addressed.

A nurse is reviewing the chart of a new client in the family medicine clinic and notes that the client is identified as "George Smith." The nurse enters the room and finds a woman in a skirt. What action by the nurse is best? a. Apologize and declare confusion about the client. b. Ask Mrs. Smith where her husband is right now. c. Ask the client about preferred forms of address. d. Explain that the chart must contain an error.

ANS: A Normal vaginal bleeding after a vaginal hysterectomy should be less than one saturated perineal pad in 4 hours. Two saturated pads in such a short time could indicate hemorrhage, which is a priority. The client with the slight temperature elevation needs to be assessed for possible infection, but not as the priority. A vaginal hysterectomy would not result in an incision the nurse could observe separating. The urinary output is normal.

A nurse receives hand-off report on four postoperative clients who each had total hysterectomies. Which client would the nurse assess first upon initial rounding? a. Vaginal hysterectomy: two saturated perineal pads in 2 hours b. Abdominal: temperature of 99° F (37.2° C), blood pressure of 116/74 mm Hg c. Vaginal: opened incisional edges and moderate bleeding d. Abdominal: urinary catheter output of 150 mL in the last 3 hours

ANS: A, B, D, E As part of client/partner education, the nurse should explain the expedited partner therapy (practice of treating both sexual partners by providing medication to the client for the partner). The nurse should also emphasize the need for abstinence from sexual intercourse until treatment is finished, proper use of condoms, and rescreening for reinfection 3 to 12 months after treatment. The use of an intrauterine device and oral contraception is not part of the plan.

A nurse wants to reduce the risk potential for transmission of chlamydia and gonorrhea with a client diagnosed with both infections. Which items should be included in the client's teaching plan? (Select all that apply.) a. Expedited partner therapy b. Abstinence until therapy is completed c. Use of intrauterine devices d. Proper use of condoms e. Rescreening for infection f. Use of oral contraception

ANS: A, B, C, D, F Common routine monitoring for this population includes lipid and liver panels, mammograms if any breast tissue is present, and PSA for natal males as the prostate is not removed during a vaginoplasty/penectomy. Cervical cancer screening is needed if the client has not had a total hysterectomy with a BSO. Renal profiles are not required based on treatment options for this population.

A nurse works with many transgender patients. What routine monitoring is important for the nurse to facilitate in this population? (Select all that apply.) a. Lipid profile b. Liver function tests c. Mammograms if breast tissue is present d. Prostate-specific antigen (PSA) for natal males e. Renal profile f. Cervical cancer screening

ANS: A, B, C, D Chlamydia, gonorrhea, syphilis, human immune deficiency virus (HIV), and acquired immune deficiency syndrome (AIDS) are all reportable to local authorities in every state. Pelvic inflammatory disease and HPV do not need to be reported.

A primary care clinic sees some clients with sexually transmitted infections. Which diseases would the nurse be required to report to the local authority? (Select all that apply.) a. Chlamydia b. Gonorrhea c. Syphilis d. Human immune deficiency virus e. Pelvic inflammatory disease f. Human papilloma virus

ANS: C A red, swollen calf could be a sign of a deep-vein thrombosis, a known adverse effect of estrogen. The nurse reports this finding immediately. The other signs and symptoms are also side effects of estrogen, but do not need to be reported as a priority.

A transgender client is taking transdermal estrogen. What assessment finding does the nurse report immediately to the primary health care provider? a. Breast tenderness b. Headaches c. Red, swollen calf d. Swollen ankles

ANS: C Spironolactone is a potassium-sparing diuretic, and hyperkalemia can cause cardiac dysrhythmias. The nurse's priority is to obtain an ECG, and then to facilitate a serum potassium level being drawn. Having the client lie down and obtaining vital signs are also important care measures, but are not the most important at this time.

A transgender client taking spironolactone is in the internal medicine clinic reporting heart palpitations. What action by the nurse takes priority? a. Draw blood to test serum potassium. b. Have the client lie down and rest. c. Obtain a STAT electrocardiogram (ECG). d. Take a set of vital signs.

C

A transgender female client reports difficulty sleeping, anxiety, and hypervigilance. She states "I just can't stop thinking about what they did to me last New Year's Eve at work. They slashed my tires. They took my purse. I see it over and over." What is the appropriate nursing action? A. Document the statements and conduct a health history. B. Look for signs of self-harm. C. Consult with the health care provider for referral to a counselor. D. Ask about drug and alcohol use.

ANS: C The client with pelvic inflammatory disease usually experiences lower abdominal tenderness. The AP can position the client. Only the nurse can administer medications and perform teaching. A heating pad, not an ice pack, is used for comfort.

A woman is admitted to the hospital for antibiotic therapy for pelvic inflammatory disease. She is in pain, with a rating of 7 on a scale of 0-10. What comfort measure can the nurse delegate to assistive personnel (AP)? a. Administer acetaminophen with codeine. b. Apply an ice pack to the lower abdomen. c. Position the client in a semi-Fowler position. d. Teach the client to increase intake of fluids.

ANS: A, D, E Alternative and complementary measures are chosen by many women. For nausea and vomiting, the best choices would be acupuncture, aromatherapy, and shiatsu. Chiropractic treatments would help pain. Journaling would be beneficial for fear and anxiety. Black cohosh is frequently used for hot flashes.

A woman is interested in alternative and complementary treatments for the nausea and vomiting caused by the side effects of chemotherapy for breast cancer. Which therapies wound the nurse suggest? (Select all that apply.) a. Acupuncture b. Chiropractic c. Journaling d. Aromatherapy e. Shiatsu f. Black cohosh

C

The nurse is teaching a client how to prevent vulvovaginitis. What information does the nurse include? A. Cleanse the inner labia daily with soap and water. B. Monthly douching may help reduce symptoms. C. Do not have unprotected sex with multiple partners. D. Wear snug-fitting polyester undergarments to prevent chafing.

ANS: D The cultural aspects of decision making need to be considered. In some cultures, the man often makes the decisions for care of the female. The woman may want to make the decision with other support people or by herself. The nurse must maintain sensitivity to cultural, religious, and personal beliefs when it comes to this personal decision. Women with a high risk for breast cancer can consider prophylactic surgery. If reconstructive surgery is considered, the procedure is more complex and will have more complications compared to a breast augmentation. There is a small risk that breast cancer can still develop in the remaining mammary tissue.

A younger woman from an unfamiliar culture is at high risk for breast cancer and is considering a prophylactic mastectomy and oophorectomy. What action by the nurse is most appropriate? a. Discourage this surgery since the woman is still of childbearing age. b. Reassure the client that reconstructive surgery is as easy as breast augmentation. c. Inform the client that this surgery removes all mammary tissue and cancer risk. d. Offer to include support people, such as the male partner, in the decision making.

Uterine Leiomyomas

Also called fibroids or myomas

D

The nurse provides health teaching for a transgender woman receiving estrogen therapy. Which statement by the client indicates a need for further teaching? A. "I'll call my doctor if I have any redness or swelling in my legs." B. "I'll have less hair on my body after taking this drug." C. "I know that the drug will make my breasts bigger." D. "I think I will have more sex drive when taking this drug."

C

The nurse uses the wrong pronoun when addressing a transgender client while taking the admitting history. What is the appropriate nursing action? A. Apologize repeatedly throughout the shift. B. Report the error to the charge nurse. C. Self-correct and continue with the admitting history. D. Ask for reassignment to another client.

Uterine Leiomyomas

Heavy bleeding, -prolonged bleeding with cycles

ANS: A Ovarian cancer usually strikes women who are middle age or older. Nulliparity, difficulty conceiving, and endometriosis all increase risk and are correct statements

The nurse has educated a community group of risk factors for ovarian cancer. Which statement by a participant shows the need for reviewing the information? a. "This is a disease of young women." b. "Never being pregnant increases my risk." c. "Difficulty conceiving is a risk factor." d. "Having endometriosis is one of the risks."

ANS: D Since the daughter is only 18, it is not recommended that she receive any of these screenings. Pap screenings are recommended to start at age 21. The HPV test is not recommended for screening but can be used for women who had an abnormal Pap test result. A mammogram is recommended for women aged 40 or older since cancers are more able to be distinguished from normal glandular tissue at that age.

The mother of an 18-year-old girl asks the nurse which screening her daughter would receive now based on evidence-based recommendations. Which suggestion by the nurse is best? a. Papanicolaou test b. Human papilloma virus (HPV) test c. Mammogram d. No screenings at this time

B

The nurse discussed gender reassignment surgery with a male-to-female client during her first visit to the office. Which client statement indicates a need for more education? A. "I will need to have at least one referral from a therapist before I have surgery." B. "The surgeon who performed my appendectomy can do my surgery." C. "I will need to take hormones for 12 months before my surgery." D. "My insurance probably won't cover this."

A, E

The nurse has delegated care for a client with a radical left mastectomy for breast cancer to assistive personnel (AP). Which AP action requires nursing intervention? Select all that apply. A. Obtains blood pressure via left arm B. Reports client's pain level to the nurse C. Applies gait belt prior to walking with the client D. Records vital signs in the electronic health record E. Assists client to administer patient-controlled analgesia (PCA)

ANS: B After vasectomy, clients are instructed to use birth control until the 3-month semen analysis shows that the procedure has worked, to use an ice pack intermittently for 24 to 48 hours, that swelling and bruising are normal, and the bandage can be removed in 48 hours. There are no sutures to be removed.

The nurse has provided postvasectomy discharge instructions to the client. What statement by the client demonstrates good understanding? a. "We can have unprotected intercourse as soon as I have healed." b. "An ice pack to my scrotum will help with the swelling." c. "I need to report signs of infection, swelling, or bruising right away." d. "The stitches can be removed here in the office in 7 to 10 days."

A

The nurse hears a patient tell her partner that condoms with spermicide are important to protect themselves from sexually transmitted infections (STI). What is the appropriate nursing response? A. Teach that spermicide has not been shown to be effective in STI prevention. B. Do nothing because the nurse should not be listening to the client's conversation. C. Educate that spermicide must be used with water-based lubricant to be effective. D. Affirm that spermicide helps to block transfer of sexually transmitted organisms.

B

The nurse is administering benzathine penicillin G intramuscularly to a client with primary syphilis. What precaution does the nurse implement when the client reports never having received penicillin before? A. Applies an ice pack to the injection site to minimize skin trauma. B. Keeps the client in the clinic for 30 minutes after drug administration. C. Administers ibuprofen to decrease the pain associated with the injection. D. Teaches to refrain from all sexual activities for a minimum of 72 hours.

ANS: A, B, D, E Bleeding, pain, masses, and discharge are common health problems that bring a client to a primary health care provider. Sexual orientation is not considered a health problem. Sexual activity would be assessed as part of the client's history.

The nurse is assessing a client for reproductive health problems. What assessments are most important? (Select all that apply.) a. Bleeding b. Pain c. Sexual orientation d. Masses e. Discharge

ANS: C Vaginal bleeding is not normal for the postmenopausal woman. Vaginal dryness and leakage of urine are common findings in adults of this age range. Pap tests may not be needed for women over 65 who have had regular cervical cancer testing with normal results.

The nurse is assessing the reproductive history of a 68-year-old postmenopausal woman. Which finding is cause for immediate action by the nurse? a. Vaginal dryness b. No Papanicolaou test for 3 years c. Bleeding from the vagina d. Leakage of urine

B

The nurse is caring for a 33-year old female client who has been intimate with women and men. What teaching will the nurse provide regarding the Gardisil-9 vaccine? A. "Patients over 26 cannot receive an HPV vaccine." B. "You will need three doses of the vaccine instead of two." C. "I will give you a single dose and you will be protected from future HPV." D. "HPV vaccines must be administered to people who have never had intercourse."

D

The nurse is caring for a client undergoing mastectomy who asks the nurse about breast reconstruction. Which of these will the nurse include in the discussion? A. Reconstruction of the nipple-areola complex is the first stage in reconstruction. B. Prostheses are not recommended because of the nature of the surgery. C. Reconstruction cannot take place until several months after a mastectomy. D. Some women want breast reconstruction using their own tissue.

B

The nurse is caring for a client who had a vaginoplasty yesterday. Which assessment finding will the nurse report to the health care provider? A. Perineal pain B. Lower extremity swelling C. Constipation D. Urinary Retention

C, D, E, F

The nurse is caring for a client who has just been diagnosed with primary syphilis. Which client statement reflects that teaching has been effective? Select all that apply. A. "I can resume having intercourse right after this injection." B. "At least this infection is not as serious as gonorrhea or chlamydia." C. "I'm afraid, but I am going to tell my partners about my diagnosis." D. "After my treatment, I still need several follow-up appointments." E. "I can take acetaminophen if I get a fever and chills after this shot." F. "I am going to wait here in the clinic 30 minutes after treatment."

A

The nurse is caring for a client who reports beginning to transition from male to female. Which nursing action is appropriate regarding pronoun use? A. Ask the patient which pronouns are preferred and use those B. Implement use of "he/him" pronouns as the client's natal sex is male C. Use "Miss or Mrs." since the client has begun the transition to female D. Document that male or female pronouns are appropriate to use at this time

A, B, D

The nurse is caring for a client who underwent a hysterosalpingogram earlier in the day. Which assessment finding will the nurse immediately report to the healthcare provider? Select all that apply. A. Fever and chills B. Heart rate 120 bpm C. Bloody vaginal discharge D. Pain in the lower quadrant E. Discomfort in the shoulder

C

The nurse is caring for a female client with uterine leiomyoma. What is the most likely assessment finding the nurse anticipates? A. Anxiety B. Pain C. Bleeding D. Constipation

ANS: D The AP is able to provide comfort through a bath. The registered nurse would review any laboratory results, complete any teaching, and assess pain and discharge.

The nurse is caring for a postoperative client following an anterior colporrhaphy. What action can be delegated to the assistive personnel (AP)? a. Reviewing the hematocrit and hemoglobin results b. Teaching the client to avoid lifting her 4-year-old grandson c. Assessing the level of pain and any drainage d. Drawing a shallow hot bath for comfort measures

D.

The nurse is caring for four clients. With which client does the nurse discuss prostate cancer screening? A. A 23 year old with a history of urinary tract infections B. A 33 year old who sustained an injury to the external genitalia C. . A 46 year old with uncontrolled hypertension D. A 57 year old who has fathered four children

A

The nurse is collecting a client history prior to a gynecologic examination. Which comment by the nurse needs to be reworded to show respect for the client's personal values and beliefs?? A. "Have you participated in homosexual relationships?" B. "How many sexual partners do you currently have?" C. "What type of protection do you use when you have sex?" D. "How often do you have intercourse?"

ANS: A Hematuria, especially at the start or end of the urine stream, could indicate infection due to possible urine retention and would cause the nurse to act promptly. Common symptoms of benign prostatic hyperplasia are urinary hesitancy, postvoid dribbling, and a weak urinary stream due to the enlarged prostate causing bladder outlet obstruction

The nurse is conducting a history on a male client to determine the severity of symptoms associated with prostate enlargement. Which finding is cause for prompt action by the nurse? a. Hematuria b. Urinary hesitancy c. Postvoid dribbling d. Weak urinary stream

A,C,D,E

The nurse is conducting a reproductive assessment of a young adult client. What assessment question will the nurse ask? (Select all that apply.) Select all that apply. A. "Have you had any sexually transmitted infections?" B. "What changes would you like to see in your appearance?" C. "If you engage in sexual activities, do you practice 'safe' sex?" D. "Are you currently experiencing any reproductive concerns?" E. "When did you first start menstruating?"

A

The nurse is counseling a client and the client's sexual partner on safer sex practices. Which client comment indicates a need for further education? A. "Condoms are for vaginal and anal sex, but not for oral sex." B. "We heard that latex condoms were better than natural membrane condoms." C. "We only use condoms once, and then throw them away." D. "If a condom breaks, we should immediately stop and replace it."

B

The nurse is counseling a client who has tested positive for syphilis. What teaching will the nurse provide regarding the client's partner? A. "You must notify your sexual partners and encourage them to seek treatment." B. "Please share accurate information about your partners so prophylactic, expedited care can be initiated." C. "Syphilis is not reportable to the local health agency and follow-up is voluntary." D. "The clinic will provide information about you to potential partners until you have completed treatment".

C

The nurse is creating a plan of care for a client with newly diagnosed pelvic inflammatory disease (PID). Which problem does the nurse identify as the client's highest priority? A. Decreased sexual drive B. Impaired self-esteem C. Bacterial infection D. Possible infertility

ANS: D In order to prevent false interpretation, the client must not douche, use vaginal medications or deodorants, or have sexual intercourse for at least 24 hours before the Pap smear. Timing is important, with the test scheduled between the client's menstrual periods so that the menstrual flow does not interfere with laboratory analysis. The specimens are placed on a glass slide and sent to the laboratory for examination and cannot be interpreted immediately.

The nurse is developing a teaching plan for a client who is scheduled for her first Papanicolaou test. What instruction by the nurse is the most accurate? a. "The timing of the Pap smear does not matter." b. "Sexual intercourse will not interfere with the results." c. "Results can be interpreted immediately in the office." d. "Results are best if you do not douche 24 hours before the test."

C

The nurse is discussing the prevention of sexually transmitted infections (STIs) with a group of young adults. What information does the nurse include? A. Spermicidal agents, when used with condoms, prevent the transmission of STIs. B. Female condoms are ineffective in preventing the transmission of STIs. C. The risk of STIs increases with the number of sexual partners. D. Use of latex condoms always keeps STIs from spreading and infecting others.

D

The nurse is discussing treatment options with a client newly diagnosed with breast cancer. Which client statement indicates a need for further teaching? A. "If I get radiation, I am not radioactive to others." B. "I might have chemotherapy before surgery." C. "Hormonal therapy is used to prevent cancer growth." D. "Radiation will remove the cancer, so I might not need surgery.

ANS: B, C, E Cryotherapy involves freezing of cervical cancer cells and is often painless. Clients are restricted from heavy lifting. They may have a heavy watery discharge for several weeks, but should report any heavy bleeding, foul-smelling drainage, or a fever. The other options are correct.

The nurse is doing home care teaching for a client who has undergone cryotherapy. Which statements by the client indicate a correct understanding of the instructions? (Select all that apply.) a. "I can resume my weight-lifting exercise class tomorrow." b. "I should not use tampons, douche, or have sexual activity." c. "I should shower rather than take a tub bath." d. "There may be a lot of bleeding for a few days." e. "There should be little or no discomfort."

C,E

The nurse is educating a 22-year-old female about the Papanicolaou (Pap) test. Which client statement indicates that further teaching is needed? (Select all that apply.) Select all that apply. A. "The specimen will be sent to a laboratory for evaluation." B. "I need to have yearly Pap tests until I am 30 years old." C. "I can have sexual intercourse the night before the test." D. "Pap smears help detect precancerous and cancerous cells." E. "I will douche the morning before I have the Pap test performed."

ANS: A Tampons need to be changed every 3 to 6 hours to avoid infection by such organisms as Staphylococcus aureus. All of the other responses are correct: use of feminine pads at night, not using tampons at all, and washing hands before tampon insertion are all strategies to prevent TSS.

The nurse is educating a client on the prevention of toxic shock syndrome (TSS). Which statement by the client indicates a lack of understanding? a. "I need to change my tampon every 8 hours during the day." b. "At night, I should use a feminine pad rather than a tampon." c. "If I don't use tampons, I should not get TSS." d. "It is best if I wash my hands before inserting the tampon.

C

The nurse is educating a group of young men about testicular self-examination (TSE). Which statement by a member of the group indicates teaching has been effective? A. "I will squeeze each testicle in my hand to feel any abnormalities." B. "I will examine my testicles right before taking a shower." C. "I will look and feel for any lumps or changes to my testes." D. "I only need to report large masses to my primary health care provider."

C

The nurse is evaluating a client's response to antibiotic treatment for pelvic inflammatory disease (PID). Which assessment finding indicates that the treatment is effective? A. Decreased vaginal discharge B. Returning libido C. Lessened pelvic tenderness D. Increased appetite

ANS: C The most appropriate question would be one that relates to benign lesions that usually change in response to hormonal changes within a menstrual cycle. Reduction of caffeine in the diet has been shown to give relief in fibrocystic breast changes, but research has not found that it has a significant impact. Questions related to the client's last mammogram or breast cancer history are not related to the nurse's assessment.

The nurse is examining a woman's breast and notes multiple small mobile lumps. Which question would be most appropriate for the nurse to ask? a. "When was your last mammogram at the clinic?" b. "How many cans of caffeinated soda do you drink in a day?" c. "Do the small lumps seem to change with your menstrual period?" d. "Do you have a first-degree relative who has breast cancer?"

A,C,D

The nurse is facilitating a discussion at an LGBTQ gathering at a college. One student asks what kind of surgeries are available for someone who wants to transition from female to male (FtM). What options will the nurse teach? (Select all that apply.) Select all that apply. A. Metoidioplasty B. Mastectomy C. Scrotoplasty D. Vaginectomy E. Penectomy

ANS: A, D, E Guidelines from the American Cancer Society include annual mammograms for low risk women starting at the age of 45 and continuing through the age of 54. At 55, women can continue annual mammography or change to every 2 years. MRI and ultrasound are done for abnormal findings or for high risk women. Breast self-awareness is important so women can detect changes early. Current data shows that SBE is not a valuable screening tool. Asymptomatic women 40 and older should have a clinical breast exam annually.

The nurse is formulating a teaching plan according to evidence-based breast cancer screening guidelines for a 50-year-old woman with low risk factors. Which diagnostic methods would be included in the plan? (Select all that apply.) a. Annual mammogram b. Magnetic resonance imaging (MRI) c. Breast ultrasound d. Breast self-awareness e. Clinical breast examination f. Self-breast examination

ANS: A,B Driving and sitting for extended periods of time should be avoided until the surgeon gives permission. For 2 to 6 weeks, exercise participation should also be avoided. All of the other responses demonstrate adequate knowledge for discharge. The client should not lift anything heavier than 10 lb (4.5 kg), should limit stair climbing, and should refrain from sexual intercourse. When intercourse is resumed, the client should use water-based lubricant and proceed slowly as the vaginal walls are tighter. This may temporarily cause some pain.

The nurse is giving discharge instructions to a client who had a total abdominal hysterectomy with a vaginal repair. Which statements by the client indicate a need for further teaching? (Select all that apply.) a. "I should not have any problems driving to see my mother, who lives 3 hours away." b. "Now that I have time off from work, I can return to my exercise routine next week." c. "My granddaughter weighs 23 lb (10.5 kg) so I need to refrain from picking her up." d. "I will have to limit the number of times that I climb our stairs at home to fewer than five times a day." e. "I need to refrain from sexual intercourse for 4 to 6 weeks." f. "When I do resume intercourse, I will use a water-based lubricant and go slowly."

D

The nurse is instructing a client with breast cancer who will be undergoing chemotherapy about the side effects of doxorubicin. Which teaching will the nurse provide? A. Report any symptoms of fatigue, cough, or edema to the oncologist. B. There are very few side effects associated with this agent. C. Doxorubicin is a type of selective estrogen receptor modulator (SERM). D. If side effects arise, they begin within 2 days of chemotherapy administration.

D

The nurse is obtaining a personal health history on a 21-year-old male. How does the nurse approach questions about his sexual practices? A. Ask if the client wants to have his partner present for the health history. B. Defer questions about his sexual practices to the health care provider. C. Skip questions about sexual practices as he is unlikely to be sexually active. D. Respect the client's choice to answer or not answer questions about sexual practices.

A

The nurse is orienting a newly hired assistive personnel (AP) to the clinic. One of the clients has self-identified as transgender. The AP states "I don't want to say the wrong thing. What do I call the person?" What is the appropriate nursing response? A. "Ask how the client would like to be addressed." B. "Look at the client's driver's license and use that." C. "Be polite and it won't matter." D. "Call the client by his or her name on the chart.

B

The nurse is performing discharge teaching for a client who is recovering from a total abdominal hysterectomy (TAH). Which client statement indicates a need for clarification? A. "I will no longer have menstrual periods." B. "I will need to find a new form of birth control." C. "I must take my temperature twice a day for the first few days after surgery." D. "I cannot jog for 2 to 6 weeks."

B,C,E

The nurse is planning care for a client who has a sexually transmitted infection (STI). Which intervention addresses the client's psychosocial needs? (Select all that apply.) Select all that apply. A. Ensure that the client's sexual partner is aware of the diagnosis. B. Refer the client to the appropriate support groups. C. Approach the client with a nonjudgmental attitude. D. Report the STI to the public health department. E. Allow the client to express fears and anxieties. F. Share experiences about working with clients with STIs.

A

The nurse is providing expedited partner treatment (EPT) to a client who is newly diagnosed with chlamydia. What is the appropriate nursing action? A. Give the drug and directions to the client for the partner. B. Provide condoms for the client and partner to use. C. Confirm the partner's diagnosis through appropriate testing. D. Make an appointment at the clinic for the partner.

ANS: C Oligospermia and azoospermia are common in clients with testicular function and can affect reproduction. The statement that there will be no effect on reproduction requires the nurse to review the information with the client. Sperm banking is an option prior to treatment to store sperm for future use. Normal sexual function is possible with one testis. Self-examination of the remaining testis is important for early detection of another tumor.

The nurse is providing preoperative education to a client prior to having an orchiectomy for testicular cancer. What statement by the client indicates the need to review the information? a. "I can still function sexually without one of my testes." b. "I will investigate sperm banking before the operation." c. "There should be no effect on my ability to reproduce." d. "Testicular self-exam will be important on the remaining testis."

ANS: B Chills and fever could indicate an infection and would be reported immediately to the primary health care provider . Mild cramping, spotting, and fatigue are normal findings after an endometrial biopsy; however, anesthesia may or may not be used.

The nurse is reviewing discharge instructions with a client who has just experienced an endometrial biopsy. Which finding would be reported to the primary health care provider immediately? a. Mild cramping b. Slight chills and fever c. Spotting of blood d. Fatigue after anesthesia

ANS: B, C, D The client would not douche, have intercourse, or use tampons until the biopsy site is healed. The client would rest for 24 hours after the procedure and would not lift heavy objects. The client would be taught to keep the perineum clean and dry by using antiseptic rinses and changes pads frequently.

The nurse is reviewing discharge plans with a client who is recovering from a cervical biopsy. Which statements indicate good understanding by the client? (Select all that apply.) a. "I can return to work this afternoon." b. "I cannot carry my toddler for 2 weeks." c. "I cannot douche until the biopsy site is healed." d. "I need to wait for about 2 weeks to have intercourse." e. "I can use a regular tampon this evening for bleeding." f. "I cannot wash my perineum for 24 hours."

ANS: A The penile implant or prosthesis is not implanted with the original phalloplasty, but months later when the original operation has healed. The other statements are accurate regarding gender-affirming surgery

The nurse is reviewing information about FtM gender-affirming surgical options with a client. What statement by the client indicates the need for further information? a. "A penile implant is inserted during the phalloplasty." b. "Vaginal atrophy can occur and lead to itching." c. "I will still need cervical cancer screening if I don't have a total hysterectomy." d. "This surgery will have many psychologic benefits for me."

ANS: A, B, C, D, E Complications from phalloplasty include wound infections, urethral complications, rectal injuries, bleeding, and donor site scarring. Recurrent urinary tract infections are not a typical complication.

The nurse is reviewing possible complications from a phalloplasty. What factors does the nurse include? (Select all that apply.) a. Wound infections b. Urethral complications c. Rectal injury d. Bleeding e. Donor site scarring f. Recurrent urinary tract infections

ANS: A, B, E, F Risk factors for breast cancer include advancing age, family and genetic history, early menarche, late menopause, postmenopausal obesity, physical inactivity, combined hormonal therapies, alcohol consumption, and lack of breast feeding.

The nurse is taking a history of a 68-year-old woman. What assessment findings would indicate a high risk for the development of breast cancer? (Select all that apply.) a. Age greater than 65 years b. Increased breast density c. Osteoporosis d. Multiparity e. Genetic factors f. Early menarche

ANS: A, B, F Smoking, multiple births, and infection with HPV are all risk factors for cervical cancer. Nulliparity is a risk factor for endometrial cancer. Poor diet could lead to decreased immunity, which is a risk, but is not directly related. Giving birth before the age of 17 is a risk factor.

The nurse is taking the history of a 24-year-old client diagnosed with cervical cancer. What possible risk factors would the nurse assess? (Select all that apply.) a. Smoking b. Multiple births c. Poor diet d. Nulliparity e. Younger than 18 at first intercourse f. Infections with HPV

C

The nurse is teaching a 19-year-old female with genital warts. Which client statement requires further education from the nurse? A. "The warts may actually disappear or resolve without any treatment at all." B. "There is no known treatment that will cure genital warts." C. "Wart remover treatment from the drugstore will help me get rid of them." D. "Genital warts may reappear at the same site."

ANS: A Fibrocystic breast changes do not increase a woman's chance of developing breast cancer. Hormone-based drugs can be used in severe cases to suppress the over-secretion of estrogen. Serious adverse effects include thrombotic events and an increased risk for uterine cancer. Limiting caffeine intake may give relief for tender breasts. The fibrocystic changes to the breasts make it more difficult to examine the breasts because of fibrotic changes and lumps.

The nurse is teaching a 45-year-old woman about her fibrocystic breast changes. Which statement by the client indicates a lack of understanding? a. "This condition will become malignant over time." b. "I understand that hormone-based drugs have serious adverse effects." c. "One cup of coffee in the morning should be enough for me." d. "This condition makes it more difficult to examine my breasts."

ANS: B, C, D, F A PDE5 inhibitor is used to treat erectile dysfunction. The client should avoid grapefruit or grapefruit juice while taking these drugs. Taking a PDE5 inhibitor along with a nitrate can cause a profound drop in blood pressure. Alcohol may interfere with the ability to have an erection. Muscle cramps, nausea, and vomiting are possible side effects if more than 1 pill a day are taken. Each medication has its own directions for how soon to take it before intercourse, from 15 minutes to 2 hours. Any PDE5 drug can lower blood pressure so the nurse alerts the client of safety precautions.

The nurse is teaching a client about side effects and adverse reactions of a PDE5 inhibitor. What information does the nurse include? (Select all that apply.) a. Refrain from eating citrus fruit within 24 hours of taking the medication. b. Stop using this drug if your primary health care provider prescribes a nitrate. c. Do not drink alcohol before having sexual intercourse. d. Muscle cramps, nausea, and vomiting are possible if you take more than 1 pill a day. e. Take this medication within 30 to 60 minutes of having sexual intercourse. f. Change positions slowly especially if you also take an anti-hypertensive drug.

A,B,C

The nurse is teaching a client how to adapt to physical and psychological changes after surgery for ovarian cancer. Which factor is included in the teaching plan? (Select all that apply.) Select all that apply. A. Encouraging the expression of grief and fears B. Suggesting the use of support groups and counseling C. Refraining from sexual intercourse for 6 weeks after surgery. D. Offering vaginal dilators E. Recommending the use of oil-based lubricants for vaginal dryness

C,D

The nurse is teaching a client how to perform breast self-examination (BSE). Which of these techniques does the nurse include in the teaching session? (Select all that apply.) Select all that apply. A. Perform the self-examination 1 week before a menstrual period. B. Teach to keep her arm by her side while performing the examination. C. Remind that a clinical breast examination and mammography are still recommended. D. Use light, medium, and then firm pressure to feel the tissue. E. A bra can be left in place during the self-examination.

ANS: A, B, D When a client is being treated with an oral antibiotic for an STI, 8 to 10 glasses of fluid should be routine, medication instructions should be reviewed, and at least a week break should occur between the antibiotic and sexual intercourse to allow for the medication's full effects if the medication was given in a single dose. Use of antacids and missing doses could decrease the effectiveness of the antibiotic.

The nurse is teaching a client who is taking an oral antibiotic for treatment of a sexually transmitted infection (STI). Which statements by the client indicate a correct understanding of the treatment? (Select all that apply.) a. "I need to drink at least eight glasses of fluid each day with my antibiotic." b. "I should read the instructions to see if I can take the medication with food." c. "Antacids should not interfere with the effectiveness of the antibiotic." d. "I need to wait 7 days after this injection to engage in intercourse." e. "It should not matter if I skip a couple of doses of the antibiotic."

ANS: C, D, E Health teaching for a client having brachytherapy would emphasize reporting abdominal pain, visible blood in the urine, and heavy vaginal bleeding. Severe diarrhea (not constipation), urethral burning, extreme fatigue, and a fever over 100° F (37.7° C) would also be reported.

The nurse is teaching a client who is undergoing brachytherapy about what to immediately report to her primary health care provider . Which signs and symptoms would be included in this teaching? (Select all that apply.) a. Constipation for 3 days b. Temperature of 99° F (37.2° C) c. Abdominal pain d. Visible blood in the urine e. Heavy vaginal bleeding f. Urinary retention

ANS: A Caffeine and alcohol have diuretic effects and so the nurse would teach about avoiding or limiting their intake. The statement about drinking wine indicates a need for further instruction. Antihistamines can cause urinary retention. Clients are taught to avoid drinking large quantities of fluid at one time.

The nurse is teaching a client with benign prostatic hyperplasia (BPH). What statement indicates the client needs further information? a. "There should be no problem with drinking wine with dinner each night." b. "I am so glad that I weaned myself off of coffee about a year ago." c. "I need to inform my allergist that I cannot take my normal antihistamine." d. "My routine of drinking a quart (liter) of water first thing in the morning needs to change."

B

The nurse is teaching a client with erectile dysfunction about taking sildenafil to achieve an erection. Which client statement demonstrates an understanding of this drug? A. "I can have sex up to 8 hours after taking the drug." B. "I might get a headache or stuffy nose when this drug is used." C. "Taking this with a drink or two of alcohol will enhance my performance." D. "If one pill doesn't work, it is acceptable for me to quickly take another pill."

C

The nurse is teaching a female client about antibiotic treatment for pelvic inflammatory disease (PID). Which client statement indicates that teaching has been effective? A. "I can resume having sex 24 hours after the first dose of antibiotics." B. "Antibiotics are always to be taken with food." C."These antibiotics must be taken for 14 days." D. "When I take the antibiotic, I also need to take an antacid."

A

The nurse is teaching a group of men about sexually transmitted infections. What sign does the nurse identify that may appear in the primary stage of syphilis? A. Painless chancre B. Urethral discharge C. Scrotal swelling D. Rash in the genital area

A

The nurse is teaching a group of young women about screening for chlamydia. Which client statement demonstrates that nursing teaching has been effective? A. "If I am a 40-year-old woman with a 'new' partner, I need to be screened again." B. "As a sexually active 19 year old, I need to be screened every 2 years." C. "At age 30, I need yearly testing, even if I have been monogamous for several years." D. "My partner got tested and treated for an STI so I don't need to be tested."

B

The nurse is teaching a group of young women. Which factor does the nurse teach increases a women's risk for development of cervical cancer? A. Having more than six pregnancies B. Having sexual intercourse at a very early age C. Using a diaphragm with spermicidal jelly for contraception D. Eating a diet that is high in fat content

ANS: B Testosterone is used as masculinizing drug therapy. Some desired effects may take up to a year to be noticed. Menses should stop within the first few months of therapy. Testosterone increases the risk of heart disease, so clients should follow a heart-healthy diet. Testosterone has several side effects, including acne, seborrhea, weight gain, edema, headaches, and possible psychosis.

The nurse is teaching a transgender client about taking testosterone. What statement by the client indicates good understanding? a. "My periods should stop immediately." b. "Some effects can take up to a year to see." c. "I am glad I don't have to watch my diet." d. "There are very few side effects since it's a normal hormone."

ANS: C Gynecomastia is enlarged breast tissue in men. It is from an enlarged ridge of glandular breast tissue and is benign. The client does not need to perform SBE nor will he undergo chemotherapy. Tamoxifen is one drug used to treat the condition, so the nurse would educate the client on the medication. Alcohol is not related.

The nurse is working with a male client who has gynecomastia. What action by the nurse is most appropriate? a. Teach the client to perform self-breast examination. b. Review the plan for chemotherapy after surgery. c. Educate him on the side effects of tamoxifen. d. Assess his usual daily alcohol intake.

C

The nurse notes bright red urinary drainage from a client who had a transurethral resection of the prostate (TURP) with continuous bladder irrigation yesterday. What is the appropriate initial nursing action? A. Calculate intake and output. B. Monitor hemoglobin and hematocrit. C. Increase the rate of the bladder irrigation. Document findings in the electronic health record

ANS: D Evidence shows that women with ovarian cancer often have recognizable, early signs such as abdominal bloating, urinary frequency or urgency, feeling full or difficulty eating, and pelvic pain. The nurse should "think ovarian" and facilitate the client having a CA-125 blood test, which is a cancer antigen test. The other actions may or may not be needed, but with these symptoms, the client needs to be evaluated for ovarian cancer.

The outpatient clinic nurse has assessed a woman who reports a month-long history of feeling full, urinary frequency, and bloating. What action by the nurse is best? a. Obtain a clean catch urine specimen. b. Instruct the client on a 3-day diet history. c. Facilitate having a pelvic ultrasound. d. Teach the woman about CA-125 test.

tertiary syphillis

Uncommon due to antibiotics, this develops 4-20 years after syphillis infection

B

Which client does the RN assess first after receiving change-of-shift report? A. A 45 year old with a history of hypothyroidism who is scheduled for a hysterectomy and bladder suspension. B. A 54 year old with an anterior and posterior colporrhaphy who has an elevated heart rate and an oral temperature of 101.2° F (38.4° C). C. A 50 year old who is receiving morphine through a client-controlled analgesia (PCA) device after a hysterectomy and who rates her pain at a level 3 (0-10 scale). D. A 48 year old who is reporting abdominal pain and light vaginal spotting after an endometrial biopsy.

A

Which client will the nurse prioritize when teaching about prostate cancer screening? A. A 50-year-old African American B. A 22-year-old Indian American C. A 30-year-old Hispanic American D. A 45-year-old Native American

B

What task does the RN delegate to assistive personnel (AP) working on the medical-surgical unit? A. Inserting a catheter in a client who has a history of uterine prolapse BAssisting with a sitz bath for a client who is postposterior colporrhaphy C. Giving report to a receiving nurse about a client who is being transferred D. Providing discharge teaching for a client who is scheduled for brachytherapy

B

What teaching will the nurse provide to a client newly diagnosed with gonorrhea? A. "Do not engage in sexual activity for 48 hours after treatment has been initiated." B. "You are contagious even if you have no outward symptoms." C. "Close follow-up is required because treatment failure is common." D. "You are only infectious while the lesions are draining."

D

What teaching will the nurse provide to a client who has just been fitted for new hearing aids? A. Leave the hearing aid on, even if not wearing it B. Immerse the ear mold in alcohol to fully clean it C. Store the hearing aid in a warm, humid bathroom when not in use D. Avoid using hair spray, makeup, and personal care products around the device

B

What will the nurse include in teaching to assist the client with BPH to address incontinence? A. Perform routine self-catheterization and bladder retraining. B. Practice Kegel exercises to improve muscle control. C. Request a prescription for sildenafil. D. Use analgesic medications as needed to maintain comfort.

A

When caring for a 28-year old healthy client, how frequently does the nurse recommend a clinical breast examination (CBE)? A. Every 3 years B. At each annual physical C. Not until age 30 as the risks are low D. To begin at age 40 when risks increase

B

When caring for four clients, which individual does the nurse identify at the highest risk for development of breast cancer? A. 33-year-old male with gynecomastia and obesity B. 45-year-old female whose mother has breast cancer C. 60-year-old male whose father died from colon cancer D. 72-year-old female who was treated for breast cancer 3 years ago

Endometriosis

When endometrial tissue forms outside the uterus. Typically in the ovaries and fallopian tubes

As soon as tests are positive

When should a patient with HIV start treatment

B

Which assessment finding in a client who recently had a right mastectomy 2 days ago will the home health nurse report to the healthcare provider? A. Temperature of 99°F B. Tingling sensation in the right arm C. Impaired range of motion in the right arm D. Drainage of 20 mL collected over 24 hours

A

Which assessment finding is the priority for the nurse to report to the health care provider? A. Painless testicular lump B. Penile discharge C. Hematuria D. Decrease increase in libido

D

Which assessment finding will the nurse report to the healthcare provider for a client who had an orchiectomy and laparoscopic radical retroperitoneal lymph node dissection this morning? A. BP 130/80, T 98.9°F, R 16, P 70 B. Urinary catheter draining clear yellow urine C. Expresses fearfulness of inability to perform sexually D. Reports pain of 9 on a 1-10 scale after receiving pain medication

C

Which client being cared for on the medical-surgical unit is appropriate for the nurse to assign to a nurse who has floated from the intensive care unit (ICU)? A. Recent radical mastectomy client requiring chemotherapy administration B. Modified radical mastectomy client needing discharge teaching C. Client with a Jackson-Pratt drain who had a mastectomy yesterday D. Stage III breast cancer client requesting information about radiation and chemotherapy

ANS: C Malignant lesions are fixed, hard and irregularly shaped and this lesion would be the priority for further diagnostic study. The other lesions are benign breast disorders. The client with nipple discharge but no palpable mass most likely has intraductal papilloma. The client who has nipple discharge but also has a mass under warm, red, edematous skin most likely has ductal ectasia.

Which finding in a female client by the nurse would receive the highest priority for further diagnostics? a. Tender moveable masses throughout the breast tissue b. Nipple discharge without a palpable mass c. Nontender fixed mass in the upper outer quadrant of the breast d. Small, painful mass under warm reddened skin and nipple discharge

B,D

Which gynecologic client does the charge nurse assign to an LPN/LVN? (Select all that apply.) Select all that apply. A. A 52 year old who just returned to the unit following a total abdominal hysterectomy. B. A 42 year old who had an abdominal hysterectomy whose primary health care provider wants to remove sutures at her bedside. C. A 48 year old who is receiving IV chemotherapy to treat stage II cervical cancer. D. A 23 year old who is nauseated after her laparotomy and needs to receive antiemetic drugs. E. A 34 year old who had a total hysterectomy for invasive cervical cancer and has a blood pressure (BP) of 88/54 mm Hg.

A,C,D,E

Which intervention does the nurse suggest for a sexually active client to reduce chances of getting an STI? (Select all that apply.) Select all that apply. A. Polyurethane condoms B. Spermicidal cream C. Abstinence D. Mutual monogamy E. Latex condoms

C, E, F

Which nursing action decreases the risk for health care disparities for transgender clients? Select all that apply. A. Refer to the client's identification card for name B. Determine gender identity based on clothing worn C. Seek to understand the experience of the transgender client D. Apologize several times if the wrong name is used for the client E. Upon meeting the client, ask what name and which pronouns are desired F. Explain how the health history and assessment are affected by gender identity

B, C, D, E

Which nursing intervention is appropriate when caring for a female client who has undergone a mastectomy and will receive chemotherapy? Select all that apply. A. Encourage client to accept her new body image B. Provide self-care resources to the primary caretaker C. Teach client about birth control options that are available D. Refer to support groups for people who have had mastectomy E. Involve partner in discussions about sexuality if client desires

B,C,F

Which sexually transmitted infection does the nurse identify that can result in a vaginal infection? (Select all that apply.) Select all that apply. A. Endometritis B. Syphilis C. Chlamydia D. Epididymitis E. Proctitis F. Gonorrhea

Stage 2

Which stage of HIV is chronic

Stage 3 Hiv

Which stage of HIV is known as aids

B,C,D

Which statement does the nurse identify as accurate regarding the prostate-specific antigen (PSA) test? Select all that apply. A. Elevated PSA levels are specific only to prostate cancer. B. African-American men may benefit from starting PSA screening at age 40. C. Health care providers may interpret the results of the PSA test differently. D. The PSA test can be used to monitor the disease course after treatment. E. PSA levels less than 7.5 ng/mL (7.5 mcg/L) may be considered normal.

gonorrhea

Which std can cause epididymitis, PID, endometriosis, salpingitis, pelvic peritonitis

A, E

Which teaching will the nurse provide to a client who has been prescribed antibiotics for pelvic inflammatory disease (PID)? Select all that apply. A. "Finish all of the prescribed drug even if you begin to feel better." B. "If you feel nauseated from the antibiotics, take a dose of Tums or Maalox." C. "Take antibiotics with food to decrease the chance of stomach irritation." D. "You may resume intercourse once you have been on the antibiotic for 48 hours." E. "You will need to return to see the health care provider after finishing drug therapy."

ANS: A The alkaline phosphatase (normal value 30 to 120 U/L [0.5 to 2.0 mckat/L]) and total calcium (normal value 9 to 10.5 mg/dL [2.25 to 2.63 mmol/L]) levels are both elevated, suggesting bone metastasis. Both the hematocrit and hemoglobin are within normal limits for females.

Which test results indicate to the nurse that some further diagnostics are needed? a. Elevated alkaline phosphatase and calcium suggests bone involvement. b. Only alkaline phosphatase is decreased, suggesting liver metastasis. c. Hematocrit and hemoglobin are decreased, indicating anemia. d. The elevated hematocrit and hemoglobin indicate dehydration

ANS: D Sexual partners, as well as the client, should be tested and treated for gonorrhea. Asking about sexually transmitted infection history, last sexual encounter, and onset of symptoms would be helpful with the history taking, but the priority is treating the client's sexual partners to limit the spread of the infection.

While evaluating a client for treatment of gonorrhea, which question is the most important for the nurse to ask? a. "Do you have a history of sexually transmitted infection?" b. "When was your last sexual encounter?" c. "When did your symptoms begin?" d. "Can you remember your partners and contact them to get treated?"

Not okay

With cART 1 missed dose is okay, what about two missed doses?

Bigger fibroid

With uterine leiyomyomas, changes in bowel movements can indicate


Conjuntos de estudio relacionados

ap european history semester 2 final

View Set

SAS Base Programing: Syntax Review

View Set

MARK TWAIN 7A EARTH SCIENCE HONORS ROCKS AND MINERALS EXAM

View Set

Section 4 Lesson 1: Supertypes and Subtypes

View Set

AutoCAD certification exam study guide

View Set

Unit 5: Agriculture and Rural Land-Use Patterns and Processes

View Set